2018 Prelims PYQ

You might also like

Download as pdf or txt
Download as pdf or txt
You are on page 1of 73

UPSC Prelims Previous Years Questions

2018

Indian Economy

Q1. As per the NSSO 70th Round "Situation Assessment Survey of Agricultural Households", consider
the following statements:

1. Rajasthan has the highest percentage share of agricultural households among its rural
households.
2. Out of the total agricultural households in the country, a little over 60 percent belong to OBCs.
3. In Kerala, a little over 60 percent of agricultural households reported to have received
maximum income from sources other than agricultural activities.

Which of the statements given above is/are correct?

(a) 2 and 3 only


(b) 2 only
(c) 1 and 3 only
(d) 1, 2 and 3

Answer: (c)

The situation assessment survey of agricultural households is conducted by the National Sample
Survey Office (NSSO). It is a detailed socio-economic survey of farmer households and is the only
large-scale survey that focuses specifically on estimating household crop income in India.

Statement 1 is correct: As per the survey, Rajasthan had the highest percentage of agricultural
households (78.4 percent) among its rural households and the least percentage stood for Kerala (27.3
percent).

Statement 2 is not correct: Out of the total estimated agricultural households in the country, about
45 percent belonged to Other Backward Classes (OBCs).

Statement 3 is correct: In Kerala, about 61 percent of the agricultural households reported to have
received maximum income from sources other than agricultural activities.
Recently, the government has cleared the formation of an overarching body- National Statistical
Office (NSO) through the merger of the NSSO and Central Statics Office (CSO) under the Ministry of
Statistics and Programme Implementation (MoSPI).

Q2. Which one of the following best describes the term "Merchant Discount Rate" sometimes seen
in news?

(a) The incentive given by a bank to a merchant for accepting payments through debit cards
pertaining to that bank.
(b) The amount paid back by banks to their customers when they use debit cards for financial
transactions for purchasing goods or services.
(c) The charge to a merchant by a bank for accepting payments from his customers through the
bank's debit cards.
(d) The incentive given by the Government to merchants for promoting digital payments by their
customers through Point of Sale (PoS) machines and debit cards.

Answer: (c)

Option (c) is correct: Merchant Discount Rate (alternatively referred to as the Transaction Discount
Rate) is the charge to a merchant by a bank for accepting payments from his customers through the
bank’s debit as well as credit cards.

Merchant Discount Rate (alternatively referred to as the Transaction Discount Rate or TDR) is the sum
total of all the charges and taxes that a digital payment entails. For instance, the MDR includes bank
charges, which a bank charges customers and merchants for allowing payments to be made digitally.
Similarly, MDR also includes the processing charges that a payments aggregator has to pay to online
or mobile wallets or indeed to banks for their service.

The rate is expressed as a percentage of transaction amount and is shared in a pre-agreed ratio
between the bank and the merchant.

Q3. With reference to India's decision to levy an equalization tax of 6% on online advertisement
services offered by non-resident entities, which of the following statements is/are correct?

1. It is introduced as a part of the Income Tax Act.


2. Non-resident entities that offer advertisement services in India can claim a tax credit in their
home country under the "Double Taxation Avoidance Agreements".

Select the correct answer using the code given below:

(a) 1 only
(b) 2 only
(c) Both 1 and 2
(d) Neither 1 nor 2

Answer: (d)

Equalisation levy of 6% was introduced in 2016 as a direct tax on non-resident service providers for
online advertisement as existing Double Tax Avoidance Agreements and other taxation provisions
could not rightfully tax the overseas digital transactions, due to which these transaction by non-
resident service providers for online advertisement largely went untaxed. Such a levy could ensure
the country gets its rightful share of taxes.

Statement 1 is not correct: Equalization tax of 6% was introduced as a separate item under the
Finance Bill 2016 and not as a part of Income Tax Act.

Statement 2 is not correct: There was a major criticism of the decision that the tax credit was not
available to the taxpayers under the "Double Taxation Avoidance Agreements" which could result
in double taxation for the non-residents.

Recent amendments to the Finance Act 2020 have expanded the net of equalisation levy to non-
resident e-commerce players also.

Q4. Consider the following statements:

1. The Fiscal Responsibility and Budget Management (FRBM) Review Committee Report has
recommended a debt to GDP ratio of 60% for the general (combined) government by 2023,
comprising 40% for the Central Government and 20% for the State Governments.
2. The Central Government has domestic liabilities of 21% of GDP as compared to that of 49% of
GDP of the State Governments.
3. As per the Constitution of India, it is mandatory for a State to take the Central Government's
consent for raising any loan if the former owes any outstanding liabilities to the latter.

Which of the statements given above is/are correct?

(a) 1 only
(b) 2 and 3 only
(c) 1 and 3 only
(d) 1, 2 and 3

Answer: (c)
FRBM Act was enacted in 2003 with an aim to introduce transparency in India’s Fiscal Management
System. It sets particular Debt-to-GDP, Revenue Deficit and Fiscal Deficit targets for the Government.
Subsequently, these targets were revised by the FRBM Review Committee set under N.K Singh.

Statement 1 is correct: The FRBM review committee in its 2017 report has recommended bringing
down the debt to GDP ratio for both Centre and States combined to 60 percent (which comprised 40
percent on account of centre and 20 percent on account of states) by 2023.

Statement 2 is not correct: At the time of the report submission domestic liabilities of the Central
Government accounted for 46.5% of the GDP and that of States accounted for 24% of the GDP.

Statement 3 is correct: Article 293 (3) of the Constitution of India states that, “A State may not
without the consent of the Government of India raise any loan if there is still outstanding any part of
a loan which has been made to the State by the Government of India or by its predecessor
Government, or in respect of which a guarantee has been given by the Government of India or by its
predecessor Government”.

The Act allows for an “escape clause” under which in special circumstances like calamities the
government can flexibly follow the Fiscal Deficit targets, allowing it for more room and space to tackle
the situation. This terminology of escape clause was innovated by the N.K Singh Panel. For the Fiscal
Year 2019-20, the total Central Government debt stood at 48.6% of the GDP.

Q5. Consider the following statements:

1. The quantity of imported edible oils is more than the domestic production of edible oils in the
last five years.
2. The Government does not impose any customs duty on all the imported edible oils as a special
case.

Which of the statements given above is/are correct?

(a) 1 only
(b) 2 only
(c) Both 1 and 2
(d) Neither 1 nor 2

Answer: (a)

Statement 1 is correct: Edible oil imports are a huge cause of concern due to their rising import bill.
A high domestic demand for edible oils in the country, makes it dependent upon imports for meeting
around 70 percent of its domestic demand. Hence, the quantity of imported edible oils is more than
the domestic production of edible oils in the last five years.

Statement 2 is not correct: The government levies 35% and 45% import duty on all crude and refined
edible oils respectively. While in 2018 the import duty on Olive oil was 40%.

Additional Information:

Around 62 percent of edible oil imports are accounted for by Palm Oil due to a high domestic demand.
With effect from January 2020, import policy of refined Palm Oil is amended from ‘free’ to
‘Restricted’ category, meaning that the government has imposed certain trade barriers which the
importer has to overcome before importing the product into the country.

Q6. Which one of the following links all the ATMs in India?

(a) Indian Banks' Association


(b) National Securities Depository Limited
(c) National Payments Corporation of India
(d) Reserve Bank of India

Answer: (c)

The Reserve Bank of India encouraged the setting up of National Payments Corporation of India
(NPCI) to act as an umbrella organisation for operating various Retail Payment Systems (RPS) in
India. NPCI became functional in early 2009. NPCI has taken over the National Financial Switch (NFS)
from the Institute for Development and Research in Banking Technology (IDRBT).

Option (c) is correct: National Payments Corporation of India links all the ATMs in India. It is expected
to bring greater efficiency by way of uniformity and standardization in retail payments and expanding
and extending the reach of both existing and innovative payment products for greater customer
convenience.

Additional Information:

National Payments Corporation of India (NPCI), an umbrella organisation for operating retail
payments and settlement systems in India, is an initiative of Reserve Bank of India (RBI) and Indian
Banks’ Association (IBA) under the provisions of the Payment and Settlement Systems Act, 2007, for
creating a robust Payment & Settlement Infrastructure in India. It has been incorporated as a “Not
for Profit” Company under the provisions of Section 25 of Companies Act 1956 (now Section 8 of
Companies Act 2013).
Q7. Consider the following statements:

1. Capital Adequacy Ratio (CAR) is the amount that banks have to maintain in the form of their
own funds to offset any loss that banks incur if the account holders fail to repay dues.
2. CAR is decided by each individual bank.

Which of the statements given above is/are correct?

(a) 1 only
(b) 2 only
(c) Both 1 and 2
(d) Neither 1 nor 2

Answer: (a)

Banks need to maintain a stipulated amount of capital as a percentage of its total credit exposure in
order to offset any losses that the banks may incur if its credit turns bad.

Statement 1 is correct: Capital Adequacy Ratio (CAR) is the ratio of a bank’s capital in relation to its
risk-weighted assets and current liabilities. It is the amount that banks have to maintain in the
form of their own funds to offset any loss that banks incur if the account holders fail to repay
dues.

Statement 2 is not correct: CAR is decided by the Reserve Bank of India (RBI) and not by each
individual bank. It is decided by the Central Bank to prevent commercial banks from taking excess
leverage and becoming insolvent in the process.

As per RBI norms, Indian Scheduled Commercial Banks are required to maintain a CAR of 9% while
Indian Public Sector Banks maintain a CAR of 12%.

Q8. Consider the following statements:

1. The Food Safety and Standards Act, 2006 replaced the Prevention of Food Adulteration Act,
1954.
2. The Food Safety and Standards Authority of India (FSSAI) is under the charge of Director
General of Health Services in the Union Ministry of Health and Family Welfare.

Which of the statements given above is/are correct?

(a) 1 only
(b) 2 only
(c) Both 1 and 2
(d) Neither 1 nor 2

Answer: (a)

The Food Safety and Standards Act was enacted in 2006 as an Act to consolidate the laws relating to
food and to establish the Food Safety and Standards Authority of India for laying down science-based
standards for articles of food and to regulate their manufacture, storage, distribution, sale, and
import, to ensure availability of safe and wholesome food for human consumption and for matters
connected therewith or incidental thereto.

Statement 1 is correct: The Food Safety and Standard Act, 2006 replaced various Acts and Orders like
the Edible Oils Packaging (Regulation) Order 1998, Fruit Products Order (FPO) 1955, Meat Food
Products Order (MFPO), Prevention of Food Adulteration Act, 1954, etc.

Statement 2 is not correct: The Ministry of Health & Family Welfare, Government of India is the
administrative Ministry for the implementation of FSSAI. The Chairperson and Chief Executive Officer
of Food Safety and Standards Authority of India (FSSAI) is appointed by the Government of India, who
holds the rank of Secretary to the Government of India. The authority acts in independent capacity.

Q9. With reference to the provisions made under the National Food Security Act, 2013, consider the
following statements:

1. The families coming under the category of 'below poverty line (BPL)' only are eligible to
receive subsidised food grains.
2. The eldest woman in a household, of age 18 years or above, shall be the head of the household
for the purpose of issuance of a ration card.
3. Pregnant women and lactating mothers are entitled to a 'take-home ration' of 1600 calories
per day during pregnancy and for six months thereafter.

Which of the statements given above is/are correct?

(a) 1 and 2 only


(b) 2 only
(c) 1 and 3 only
(d) 3 only

Answer: (b)

The National Food Security Act, 2013 provides a legal right to persons belonging to “eligible
households” to receive food grains at a subsidised price. It has the objective to provide for food and
nutritional security in the human life cycle approach, by ensuring access to adequate quantities of
quality food at affordable prices to people to live a life with dignity.

Statement 1 is not correct: The Act provided for coverage of up to 75% of the rural and 50% of the
urban population to receive highly subsidized food grains under Targeted Public Distribution System.
Under this Act, two-thirds of the total population of the country or approximately 81 crore persons
are estimated to receive benefits under the Targeted Public Distribution System. Hence, the families
coming under the category of 'below poverty line (BPL)' are not the only eligible people to receive
subsidised food grains.

Statement 2 is correct: As a step towards women empowerment, the eldest woman of the household
of age 18 years or above is mandated to be the head of the household for the purpose of issuing
ration cards under the Act.

Statement 3 is not correct: One of the guiding principles of the Act is its life-cycle approach wherein
special provisions have been made for pregnant women and lactating mothers and children in the
age group of 6 months to 14 years. Pregnant women and lactating mothers are entitled to a nutritious
"take home ration" of 600 Calories and a maternity benefit of at least Rs 6,000 for six months.

Q10. India enacted the Geographical Indications of Goods (Registration and Protection) Act, 1999
in order to comply with the obligations to

(a) ILO
(b) IMF
(c) UNCTAD
(d) WTO

Answer: (d)

Option (d) is correct: A geographical indication (GI) is a sign used on products that have a specific
geographical origin and possess qualities or a reputation that are due to that origin. India enacted the
Geographical Indications of Goods (Registration and Protection) Act, 1999 in order to comply with
the obligations to the World Trade Organization.

Trade-Related Aspects of Intellectual Property Rights (TRIPS) Agreement was signed in 1994, forming
a part of the larger Marrakesh Agreement forming the WTO. Geographical Indications have been
understood to be Intellectual Property Rights under the TRIPS Agreement.

Saffron of Jammu & Kashmir is the recent item which has received GI tag in India.
Q11. With reference to digital payments, consider the following statements:

1. BHIM app allows the user to transfer money to anyone with a UPI-enabled bank account.
2. While a chip-pin debit card has four factors of authentication, BHIM app has only two factors
of authentication.
Which of the statements given above is/are correct?

(a) 1 only
(b) 2 only
(c) Both 1 and 2
(d) Neither 1 nor 2

Answer: (a)

Digital payment is a transaction that takes place via digital or online modes using Mobile or
Computer, without physical exchange of money involved. Here the payer and the payee, use electronic
mediums to exchange money.

Statement 1 is correct: Bharat Interface for Money (BHIM) is mobile Payment app developed by the
National Payments Corporation of India (NPCI) based upon the system of Unified Payments Interface
(UPI). BHIM app integrates the Payments platform as the official app of the government of India. It
allows the user to transfer money to anyone with a UPI-enabled bank account.

Statement 2 is not correct: Debit cards do not have four factor authorization and BHIM app has three
factor authorization. A system that uses two-factor authentication is a bank account with a debit
card, where you need to know a PIN and have the physical debit card to withdraw money. A two-
factor authentication app is a similar idea, but instead of a physical card, the second element is your
phone. The BHIM app has three levels of authentication, First the app binds with a device’s ID and
mobile number, second a user needs to sync whichever bank account (UPI or non-UPI enabled) in
order to conduct the transaction. Third, when a user sets up the app, they are asked to create a pin
which is needed to log into the app.

Q12. International Labour Organization's Conventions 138 and 182 are related to

(a) Child Labour


(b) Adaptation of agricultural practices to global climate change
(c) Regulation of food prices and food security
(d) Gender parity at the workplace

Answer: (a)

Option (a) is correct: The two International Labour Organization's (ILO) Conventions on child labour
are Convention No. 138 on Minimum Age and Convention No. 182 on the Worst Forms of Child
Labour. These Conventions are “fundamental” Conventions. This means that, under the ILO
Declaration on Fundamental Principles and Rights at Work, all ILO member States have an obligation
to respect, promote and realize the abolition of child labour, even if they have not ratified the
Conventions in question.

ILO Convention No. 182 is the first ILO Convention to achieve universal ratification. It was also the
most rapidly ratified Convention in the history of the ILO, with the majority of ratifications occurring
within the first 3 years after it was adopted in 1999. ILO Convention No. 138 has also been widely
ratified by ILO member States.

Q13. Consider the following countries:

1. Australia
2. Canada
3. China
4. India
5. Japan
6. USA

Which of the above are among the 'free-trade partners' of ASEAN?

(a) 1, 2, 4 and 5 only


(b) 3, 4, 5 and 6 only
(c) 1, 3, 4 and 5 only
(d) 2, 3, 4 and 6 only

Answer: (c)

The Association of Southeast Asian Nations (ASEAN) was established in August 1967 in Bangkok,
Thailand, with the signing of the ASEAN Declaration (Bangkok Declaration). Currently, it has 10
member nations which are: Brunei, Cambodia, Indonesia, Laos, Malaysia, Myanmar, the Philippines,
Singapore, Thailand, and Vietnam.
Option (c) is correct: ASEAN has a total of 6 free trade partners namely: Australia, New Zealand, South
Korea, Japan, India, and China. ASEAN and its 5 free trade partners (excluding India) have entered
into a regional trading agreement called the Regional Comprehensive Economic Partnership (RCEP).
The purpose of RCEP was to make it easier for products and services of each of these countries to be
available across this region. India has however chosen to stay out of RCEP.

Q14. Which one of the following statements correctly describes the meaning of legal tender money?

(a) The money which is tendered in courts of law to defray the fee of legal cases.
(b) The money which a creditor is under compulsion to accept in settlement of his claims.
(c) The bank money in the form of cheques, drafts, bills of exchange, etc.
(d) The metallic money in circulation in a country.

Answer: (b)

Currency notes and coins (fiat money) do not have intrinsic value like gold or silver coins, but they
cannot be refused by any citizen of the country for settlement of any kind of transaction, hence called
legal tenders.

Option (b) is correct: Legal tender money is the money which a creditor is under compulsion to accept
in settlement of his claims. Legal tenders are the mediums to settle financial obligations and
transactions which are recognised by the law of the land.

Cheques drawn on savings or current accounts, however, can be refused by anyone as a mode of
payment. Hence, demand deposits are not legal tenders.

Q15. If a commodity is provided free to the public by the Government, then

(a) the opportunity cost is zero.


(b) the opportunity cost is ignored.
(c) the opportunity cost is transferred from the consumers of the product to the tax-paying
public.
(d) the opportunity cost is transferred from the consumers of the product to the Government.

Answer: (c)

Opportunity cost is the loss or gain of making a decision. It is the value of what one loses when
choosing between two or more options or alternatives. Due to the limited number of resources
available to any individual/society/country etc. choices have to be made in respect of the
benefits/uses that have to be availed out of the available resources. While choosing one use for the
available resources we forgo the other alternative uses which could have also been chosen instead of
the currently chosen use. The value that could have been obtained out of other alternative uses and
is now foregone is referred to as the opportunity cost of using one particular alternative.

Option (c) is correct: If a commodity is provided free to the public by the Government, then the
opportunity cost is transferred from the consumers of the product to the tax-paying public.

The government while providing free commodities to certain consumers uses the tax revenue that it
has obtained. The taxpayers could have also availed other benefits out of their paid taxes, hence the
opportunity cost of providing a free commodity is in the sense of other prospective benefits that could
have been availed, foregone by the tax-paying public.

Q16. Increase in absolute and per capita real GNP do not connote a higher level of economic
development, if

(a) industrial output fails to keep pace with agricultural output.


(b) agricultural output fails to keep pace with industrial output.
(c) poverty and unemployment increase.
(d) imports grow faster than exports.

Answer: (c)

Per capita real GNP accurately captures the economic growth of a nation by adjusting for price level
changes so that the actual growth in the production of goods and services can be understood. By
dividing real GNP on a per capita basis, a clearer comparison can be made with other nations having
different populations.

But, what these measures cannot accurately capture is development. Development happens when
the fruits of economic growth are translated into increased prosperity and better standards of living
for all the residents of a nation.

Option (c) is correct: Increasing poverty and unemployment reflect that the fruits of growth in Real
GNP have not adequately spread to all. While economic growth is happening for the nation in general
a section of people is left behind hence a higher level of economic development is not taking place
in the country.

Q17. Consider the following statements:

Human capital formation as a concept is better explained in terms of a process which enables

1. individuals of a country to accumulate more capital.


2. increasing the knowledge, skill levels and capacities of the people of the country.
3. accumulation of tangible wealth.
4. accumulation of intangible wealth

Which of the statements given above is/are correct?

(a) 1 and 2 only


(b) 2 only
(c) 2 and 4 only
(d) 1, 3 and 4 only

Answer: (c)

Human Capital can be defined as the economic value of an individual’s skills, abilities, knowledge, or
experience, etc.

Statement 1 and statement 3 are not correct: Human Capital Formation relates to the enhancement
in the abilities of humans as a resource in himself/herself and not the accumulation of tangible wealth
like land, money, etc. Tangible wealth is related to assets which can be seen and touched like gold,
land etc. However, these form the part of economic capital and not human capital.

Statement 2 is correct: By increasing the knowledge, skill levels and capacities of the people of the
country their ability to translate their capabilities into economic capital is enhanced thus aiding the
process of Human Capital Formation in the country.

Statement 4 is correct: Through the process of Human Capital Formation the people can accumulate
knowledge and skill levels which are attributed to intangible wealth as they cannot be touched and
seen.

Q18. Despite being a high saving economy, capital formation may not result in significant increase
in output due to

(a) weak administrative machinery


(b) illiteracy
(c) high population density
(d) high capital-output ratio

Answer: (d)

Under the concept of circular flow of income, it is illustrated that the savings from the household
sector are subsequently moved to the producers in an economy who in turn invest it and enhance the
capital formation which make available capital goods to the producers like machines, tools etc. This
aids in further production of goods translating into higher output in the economy.

Option (d) is correct: Capital-output ratio is the amount of capital needed to produce one unit of
output. A higher ratio means that more capital is needed to produce one unit of output due to which
the increased capital formation may not result in significant increase in output. A higher ratio
generally indicates inefficient deployment of the capital resources of a nation.

Q19. Consider the following statements:

1. The Reserve Bank of India manages and services Government of India Securities but not any
State Government Securities.
2. Treasury bills are issued by the Government of India and there are no treasury bills issued by
the State Governments.
3. Treasury bills offer are issued at a discount from the par value.

Which of the statements given above is/are correct?

(a) 1 and 2 only


(b) 3 only
(c) 2 and 3 only
(d) 1, 2 and 3

Answer: (c)

Government Securities are tradable instruments issued by the Central Government or the State
Governments. They acknowledge the government’s debt obligation.

Statement 1 is not correct: The Reserve Bank of India (RBI) manages and services the government
securities and debt for the centre as well as State Government Securities in terms of the agreements
that it enters in with other State Governments.

Statement 2 is correct: Treasury bills or T-bills, which are money market instruments and are short
term debt instruments issued by the Government of India. Presently, they are issued in three tenors,
namely, 91day, 182 day and 364 day. They are not issued by the State Governments.

Statement 3 is correct: Treasury bills are zero coupon securities and pay no interest. Instead, they are
issued at a discount and redeemed at the face value at maturity.

Currently, the RBI acts as banker to all the State Governments in India (including Union Territory of
Puducherry), except Sikkim. For Sikkim, it has limited agreement for management of its public debt.
Q20. With reference to Pradhan Mantri Kaushal Vikas Yojana, consider the following statements:

1. It is the flagship scheme of the Ministry of Labour and Employment.


2. It, among other things, will also impart training in soft skills, entrepreneurship, financial and
digital literacy.
3. It aims to align the competencies of the unregulated workforce of the country to the
National Skill Qualification Framework.

Which of the statements given above is/are correct?

(a) 1 and 3 only


(b) 2 only
(c) 2 and 3 only
(d) 1, 2, and 3

Answer: (c)

Statement 1 is not correct: Pradhan Mantri Kaushal Vikas Yojana is the flagship scheme of the Ministry
of Skill Development & Entrepreneurship (MSDE) implemented by National Skill Development
Corporation. The objective of this Skill Certification Scheme is to enable a large number of Indian
youths to take up industry-relevant skill training that will help them in securing a better livelihood.

Statement 2 is correct: The training programme under the scheme comes under the National Skill
Qualification Framework (NSQF) wherein various training centres provide skills and training in areas
such as soft skills, entrepreneurship, financial and digital literacy.

Statement 3 is correct: Recognition of Prior Learning is a subcomponent under the scheme wherein
the existing competencies of the individuals through the way of their knowledge/ skill set and
experience which have been gained either through formal or informal means is assessed and
recognised. The scheme aims to align the competencies of the unregulated workforce of the country
to the National Skill Qualification Framework.

Q21. With reference to the governance of public sector banking in India, consider the following
statements:

1. Capital infusion into public sector banks by the Government of India has steadily increased in
the last decade.
2. To put the public sector banks in order, the merger of associate banks with the parent State
Bank of India has been affected.

Which of the statements given above is/are correct?

(a) 1 only
(b) 2 only
(c) Both 1 and 2
(d) Neither 1 nor 2

Answer: (b)

The purpose of capital infusion into public sector banks (PSBs) is to strengthen the lending capacity
and improve credit growth among PSBs that are saddled with a heavy, non-performing assets (NPAs)
as well as are struggling to meet capital adequacy ratio.

Statement 1 is not correct: Capital infusion into public sector banks by the Government of India has
not steadily increased in the last decade. As per the Department of Financial Services data, Rs. 25,000
crores were infused in 2016-2017 Financial Year while only Rs. 10,000 crores were infused in the FY
of 2017-18.

Statement 2 is correct: In 2017, State Bank of India merged with five of its associate banks, which
are: State Bank of Bikaner and Jaipur, State Bank of Mysore, State Bank of Travancore, State Bank of
Hyderabad, and State Bank of Patiala along with Bharatiya Mahila Bank. To put the public sector
banks in order, the merger of these associate banks with the parent State Bank of India has been
affected. This step has aided in the banks rationalising its costs and enhancing its depositor base.

Q22. Consider the following items:

1. Cereal grains hulled


2. Chicken eggs cooked
3. Fish processed and canned
4. Newspapers containing advertising material

Which of the above items is/are exempted under GST (Good and Services Tax)?

(a) 1 only
(b) 2 and 3 only
(c) 1, 2 and 4 only
(d) 1, 2, 3 and 4
Answer: (a)

Goods and Services Tax was introduced in 2017 as a destination based indirect tax. The Constitution
was amended under the 101st Constitutional Amendment Act to insert Article 246A and Article 269A
among other changes to facilitate the GST.

Option (a) is correct: Cereal grains hulled, chicken eggs, and fish were exempted under the GST.
Since, chicken eggs cooked can be made available in the restaurants, these are not exempt from the
GST. Also, GST is chargeable at 5% and 18% rates in different cases on provision of advertising services
in the newspapers.

Polity and Governance

Q1. Which of the following are regarded as the main feature of “Rule of Law”?

1. Limitation of power
2. Equality before the law
3. People’s responsibility to the Government
4. Liberty and civil rights

Select the correct answer using the codes given below:

(a) 1 and 3 only


(b) 2 and 4 only
(c) 1, 2 and 4 only
(d) 1, 2, 3 and 4

Answer: (c)

The Rule of Law is the foundational feature of all modern democratic nations. A.V. Dicey, the British
jurist, has given a few elements or aspects of Rule of Law.

Option (c) is correct: Equality before the law, that is, equal subjection of all citizens (rich or poor, high,
or low, official, or non-official) to the ordinary law of the land administered by the ordinary law courts.

Absence of arbitrary power/ limitation of power, that is, no man can be punished except for a breach
of law. It implies the government has limited powers that it derived from the law.
The primacy of the rights, liberty and civil rights, of the individual, that is, the Constitution is the
result of rights of individuals as defined and enforced by the court of law rather than the
Constitution being the source of individual rights.

The first and second elements are applicable to the Indian system, the Constitution is the source of
individual rights. Rule of law along with its three basic elements results in the formation of a
government that is responsible to the people, not the other way around.

Q2. Which one of the following reflects the most appropriate relationship between law and liberty?

(a) If there are more laws, there is less liberty.


(b) If there are no laws, there is no liberty.
(c) If there is liberty, laws have to be made by the people.
(d) If laws are changed too often, liberty is in danger.

Answer: (b)

Liberty or freedom is formally defined as absence of restraint. Law, on the other hand, imposes
restraints on certain activities of individuals. When liberty is accepted as a universal principle, it is
imperative that liberty of one individual does not endanger the liberty of another individual.

Option (b) is correct: This necessitates restriction on liberty of individuals so as to allow equal liberty
to all. Law is precisely the means to impose such restrictions. Law restricts liberty in requiring us to
do things that we might otherwise want to do, and in requiring us to refrain from actions that we
might otherwise want to do. Therefore, restrictions upon liberty imposed by the law intend to
protect the liberty of others which would be impeded by the behaviour that the law restricts. Thus,
if there are no laws, there is no liberty.

Q3. Consider the following statements:

1. In the first Lok Sabha, the single largest party in the opposition was the Swatantra Party.
2. In the Lok Sabha, a “Leader of the Opposition” was recognised for the first time in 1969.
3. In the Lok Sabha, if the party does not have a minimum of 75 members, its leader cannot be
recognised as the leader of opposition.

Which of the following statements given is/are correct?

(a) 1 and 3 only


(b) 2 only
(c) 2 and 3 only
(d) 1, 2 and 3

Answer: (b)

Statement 1 is not correct: The first Lok Sabha elections in India were held in 1951-52. Indian National
Congress won with the sweeping majority scoring 364 out of 489 seats. The Communist Party of India
emerged as the largest party in the opposition winning a total of 16 seats. The Swatantra Party on
the other hand was formed in 1959. It was founded by C. Rajagopalachari who was dissatisfied with
the Indian National Congress’s socialist agenda.

Statement 2 is correct: In parliamentary form government the Leader of the Opposition has a
significant role to play. His function involves providing constructive criticism of government policies.
In 1969, when Indira Gandhi was the Prime Minister, the Congress split to form the Indian National
Congress (Requisitionists) and the Indian National Congress (Organisation). The Leader of INC(O), Ram
Subhag Singh, became the first person to be formally recognised as LoP in the Lok Sabha. Leaders of
Opposition in the Lok Sabha and Rajya Sabha were accorded statutory recognition in 1977.

Statement 3 is not correct: The leader of the largest opposition party having not less than one-tenth
seats of the total strength of the house is recognised as the Leader of the Opposition in that house.

Q4. Consider the following statements:

1. Parliament of India can place a particular law in the Ninth Schedule of the Constitution of
India.
2. The validity of law placed in the Ninth Schedule cannot be examined by any court and no
Judgement can be made on it.

Which of the statements given above is/are correct?

(a) 1 only
(b) 2 only
(c) 1 and 2 only
(d) Neither 1 nor 2

Answer: (a)

Statement 1 is correct: The Ninth Schedule contains a list of central and state laws. The Schedule
became a part of the Constitution in 1951 when the document was amended for the first time. It
was created by the new article 31B, which along with 31A were inserted in the Constitution to
protect laws related to agrarian reforms and for abolishing the Zamindari system. At present there
are around 284 laws in the Ninth Schedule. Thus, the Parliament can place a particular law in the
Ninth Schedule of the Constitution of India.

Statement 2 is not correct: In a significant judgement delivered in I.R. Coelho case (2007), the
Supreme Court ruled that there could not be any blanket immunity from judicial review of laws
included in the Ninth Schedule. The court held that judicial review is a ‘basic feature’ of the
Constitution, and it could not be taken away by putting a law in the Ninth Schedule.

Q5. If the president of India exercises his power as provided under Article 356 of the constitution in
respect of a particular state, then

(a) the assembly of a state is automatically dissolved.


(b) the power of the Legislature of that State shall be exercisable by or under the authority of
the Parliament.
(c) Article 19 is suspended in that State.
(d) the President can make laws relating to that State.

Answer: (b)

The President's Rule can be proclaimed under Article 356 on two grounds, one mentioned in Article
356 itself and another in Article 365. When the President's rule is imposed, the President dismisses
the State Council of Ministers headed by the Chief Minister. Further, the President either suspends
or dissolves the State Legislative Assembly.

Option (b) is correct: When the State Legislature is suspended or dissolved, the power of the
Legislature of that State shall be exercisable by or under the authority of the Parliament. The
Parliament can also delegate the power to make laws for the State to the President or to any other
Authority specified by him in this regard.

Q6. Consider the following statements:

1. The Speaker of the Legislative Assembly shall vacate his/her office if he/she ceases to be a
member of the Assembly.
2. Whenever the Legislative Assembly is dissolved, the Speaker shall vacate his/her office
immediately.

Which of the statements given above is/are correct?

(a) 1 only
(b) 2 only
(c) Both 1 and 2
(d) Neither 1 nor 2

Answer: (a)

Statement 1 is correct: The Speaker is elected by the assembly itself from amongst its members.
Usually, the Speaker remains in office during the life of the Assembly. However, he/she vacates
his/her office earlier if he ceases to be a member of the Assembly, if he resigns by writing to the
deputy speaker, or if he is removed by a resolution passed by a majority of all the then members of
the assembly. Such a resolution can be moved only after giving 14 days advance notice.

Statement 2 is not correct: Whenever the Legislative Assembly is dissolved, the Speaker does not
vacate his office and continues till the newly elected Legislative Assembly meets.

Q7. With reference to the Parliament of India, which of the following Parliamentary Committees
scrutinizes and reports to the House whether the powers to make regulations, rules, sub-rules, by-
laws, etc. conferred by the Constitution or delegated by the Parliament are being properly exercised
by the Executives within the scope of such delegation?

(a) Committee on Government Assurances


(b) Committee on Subordinate Legislation
(c) Committee on rules
(d) Business Advisory Committee

Answer: (b)

Parliament performs varied, complex, and voluminous functions. But it neither has the adequate time
nor necessary expertise to make a detailed scrutiny of all legislative measures and other matters.
Therefore, it is assisted by a number of Parliamentary Committees in the discharge of its duties.

Option (b) is correct: The Committee on Subordinate Legislation examines and reports to the House
whether the powers to make regulations, rules, sub-rules delegated by the Parliament or conferred
by the Constitution to the Executive are being properly exercised by it. In both the Houses, the
Committee consists of 15 members. It was constituted in 1953.

Q8. Consider the following statements:

1. No criminal proceedings shall be instituted against the Governor of a State in any court
during his term of office.
2. The emoluments and allowances of the Governor of a state shall not be diminished during
his term of office.
Which of the statements given above is/are correct?

(a) 1 only
(b) 2 only
(c) Both 1 and 2
(d) Neither 1 nor 2

Answer: (c)

Statement 1 is correct: The Governor is entitled to a number of privileges and immunities. He enjoys
personal immunity from legal liability for his official acts. During his term of office, he is immune
from any criminal proceedings, even in respect of his personal acts and cannot be arrested or
imprisoned. However, after giving two months’ notice, civil proceedings can be instituted against him
during his term of office in respect of his personal acts.

Statement 2 is correct: A Governor holds office for a term of five years from the date on which he
enters upon his office. He is entitled to such emoluments, allowances and privileges as may be
determined by Parliament. His emoluments and allowances cannot be diminished during his term of
office.

Q9. With reference to the election of the President of India, consider the following statements:

1. The value of the vote of each MLA varies from State to State.
2. The value of vote of MPs in Lok Sabha is more than the value of the vote of MPs of the Rajya
Sabha.

Which of the statements given above is/are correct?

(a) 1 only
(b) 2 only
(c) Both 1 and 2
(d) Neither 1 nor 2

Answer: (a)

Statement 1 is correct: The President is elected by an electoral college, which comprises only of
elected representatives (Parliament, Legislative Assemblies of States and UTs of Delhi and
Puducherry). Nominated members of Parliament are not allowed to vote. The value of a vote of an
MLA varies from State to State, mainly to reflect the population of each state. For example: the value
of one MLA’s vote in Delhi is 58, in Uttar Pradesh it is 208 and in Sikkim it is just 7.
Statement 2 is not correct: The value of vote of each Member of Parliament is equal whether he/she
is from Lok Sabha or Rajya Sabha.

Q10. Right to Privacy is protected as an intrinsic part of Right to Life and Personal Liberty. Which of
the following in the Constitution of India correctly and appropriately imply the above statement?

(a) Article 14 and the provisions under the 42nd amendment to the Constitution.
(b) Article 17 and the Directive Principle of the State Policy in Part IV
(c) Article 21 and the freedoms guaranteed in Part III
(d) Article 24 and provisions under the 44th Amendments to the Constitution

Answer: (c)

Option (c) is correct: The Supreme Court in the case of Puttaswamy vs Union of India (2017) delivered
the landmark judgement where it upheld the Right to Privacy as a Fundamental Right protected under
Part III of the Indian Constitution. The Supreme Court held that “Right to Privacy” is an integral part
of Right to Life and Personal Liberty guaranteed in Article 21 of the Constitution.

The Right to Privacy was reinforced by the concurring opinions of the judges who recognized that the
Right to Privacy includes autonomy over personal decisions, as in beef consumption; bodily integrity,
as in reproductive rights; and protection of personal information, as in the privacy of health records.

Q11. In the federation established by the Government of India Act of 1935, residuary powers given
to the

(a) Federal Legislature


(b) Governor General
(c) Provincial Legislature
(d) Provincial Governors

Answer: (b)

Option (b) is correct: The Government of India Act of 1935 provided for the establishment of an All-
India Federation consisting of Provinces and Princely States as units. The Act divided the powers
between the Centre and units in terms of three lists, Federal List (for Centre, with 59 items),
Provincial List (for provinces, with 54 items) and the Concurrent List (for both, with 36 items).
Residuary powers were given to the Governor General.

The Government of India Act of 1935 abolished dyarchy in the provinces and introduced ‘provincial
autonomy’ in its place and introduced bicameralism in six out of eleven provinces. Thus, the
legislatures of Bengal, Bombay, Madras, Bihar, Assam and the United Provinces were made bicameral
consisting of a legislative council (upper house) and a legislative assembly (lower house), further
extended the principle of communal representation by providing separate electorates for depressed
classes (scheduled castes), women and labour (workers). It abolished the Council of India, established
by the Government of India Act of 1858. The secretary of state for India was provided with a team of
advisors. It provided for the establishment of a Reserve Bank of India to control the currency and
credit of the country and provided for the establishment of not only a Federal Public Service
Commission but also a Provincial Public Service Commission and Joint Public Service Commission for
two or more provinces.

Q12. Regarding Money Bill, which of the following statements is not correct?

(a) A bill shall be deemed to be a Money Bill if it contains only provisions related to imposition,
abolition, remission, alteration or regulation of any tax.
(b) A Money Bill has the provisions for the custody of the Consolidated Fund of India or the
Contingency Fund of India.
(c) A Money Bill is concerned with the appropriation of moneys out of the Contingency Fund of
India.
(d) A Money Bill deals with the regulations of borrowings of money or giving of any guarantee
by the Government of India.

Answer: (c)

Option (c) is correct: Article 110 of the Constitution deals with the definition of Money Bills. It states
that a bill is deemed to be a Money Bill if it contains ‘only’ provisions dealing with all or any of the
following matters:

● The imposition, abolition, remission, alteration, or regulation of any tax.


● The regulation of the borrowing of money by the Union government.
● The custody of the Consolidated Fund of India or the Contingency Fund of India, the payment
of moneys into or the withdrawal of money from any such fund.
● The appropriation of money out of the Consolidated Fund of India.
● Declaration of any expenditure charged on the Consolidated Fund of India or increasing the
amount of any such expenditure.
● The receipt of money on account of the Consolidated Fund of India or the public account of
India or the custody or issue of such money, or the audit of the accounts of the Union or of a
state.
● Any matter incidental to any of the matters specified above.
Environment and Geography

Q1. Which of the following leaf modifications occur(s) in the desert areas to inhibit water loss?

1. Hard and waxy leaves


2. Tiny leaves
3. Thorns instead of leaves

Select the correct answer using the code given below:

(a) 2 and 3 only


(b) 2 only
(c) 3 only
(d) 1, 2 and 3

Answer: (d)

Option (d) is correct: Desert plants lose very little water through transpiration. The leaves in desert
plants are either absent, very small (tiny), or they are present in the shape of spines. This helps in
reducing loss of water from the leaves through transpiration. plants growing in extreme dry conditions
throughout the year adopted thorns instead of leaves to prevent loss of water.

Photosynthesis in these plants is usually carried out by the stems. The stem is also covered with a thick
waxy layer, which helps to retain water. Hence, they have hard and waxy leaves as well. Most desert
plants have roots that go very deep into the soil for absorbing water.

Xerophytes are plants growing in extreme dry conditions throughout the year. For example, plants
growing in deserts (psammophytes), on rock (lithophytes) or alpine plants growing above 14000 feet
altitude.

Q2. How is the National Green Tribunal (NGT) different from the Central Pollution Control Board
(CPCB)?

1. The NGT has been established by an Act whereas the CPCB has been created by an executive
order of the Government.
2. The NGT provides environmental justice and helps reduce the burden of litigation in the higher
courts whereas the CPCB promotes cleanliness of streams and wells, and aims to improve the
quality of air in the country.

Which of the statements given above is/are correct?

(a) 1 only
(b) 2 only
(c) Both 1 and 2
(d) Neither 1 nor 2

Answer: (b)

Statement 1 is not correct: The National Green Tribunal has been established under the National
Green Tribunal Act 2010 for effective and expeditious disposal of cases relating to environmental
protection and conservation of forests and other natural resources including enforcement of any legal
right relating to environment and giving relief and compensation for damages to persons and
property and for matters connected therewith or incidental thereto. Whereas, the Central Pollution
Control Board (CPCB), a statutory organisation, constituted in September 1974 under the Water
(Prevention and Control of Pollution) Act, 1974. Further, CPCB was entrusted with the powers and
functions under the Air (Prevention and Control of Pollution) Act, 1981.

Statement 2 is correct: The National Green Tribunal's dedicated jurisdiction in environmental


matters provides speedy environmental justice and helps reduce the burden of litigation in the
higher courts. Principal Functions of the CPCB, as spelt out in the Water (Prevention and Control of
Pollution) Act, 1974, and the Air (Prevention and Control of Pollution) Act, 1981, is to promote
cleanliness of streams and wells in different areas of the States by prevention, control, and abatement
of water pollution, and to improve the quality of air and to prevent, control or abate air pollution in
the country.

Q3. Which of the following has/have shrunk immensely/dried up the recent past due to human
activities?

1. Aral Sea
2. Black Sea
3. Lake Baikal

Select the correct answer using the code given below:


(a) 1 only
(b) 2 and 3 only
(c) 2 only
(d) 1 and 3 only

Answer: (a)

Option (a) is correct: Decades ago, the Aral Sea was the world's fourth largest body of inland water,
but it has shrunk immensely due to diversion of rivers for irrigation (human activities). Two decades
ago, it split into the small northern and larger southern sections, and further divisions have been
happening ever since. Moreover, in the 1960s, the Soviet Union undertook major water diversion
projects on the Syr Darya and Amu Darya rivers, capturing water that once fed into the Aral Sea.

Dust blowing from the exposed lakebed eventually degraded the soils, forcing further water diversion
efforts to revive them. On a larger scale, loss of the Aral Sea's water influenced regional climate,
making the winters even colder and the summers much hotter. Fifty years later, the lake is virtually
gone.

Black Sea is often seen in the news in the context of Russian affairs. Past 60 years, the oxygen-rich
top layer of the Black Sea decreased from 140 meters to 90 meters deep, which amounts to an over
40 percent dip in habitable waters. Two existing causes behind the shrinkage: an abundance of
nutrients, particularly algae that led to great consumption of oxygen and global warming. With
warmer winters, there is a lower volume of dense water created and this lowers the oxygen content.
Warmer waters also mean the sea is now able to accumulate less dissolved gas, including oxygen.
The Black Sea has lost more than a third of its habitable volume. The Habitable area in Black Sea is
shrinking, but the actual volume is not shrinking immensely.

In 2015, Russia officially declared Lake Baikal an emergency zone because of a catastrophic drop in
its water level. Baikal is the globe’s largest freshwater lake and is on the UNESCO World Heritage list.
Industrial area of the closed Baikal Pulp and Paper Mill that has been polluting the lake for decades.
building more hydroelectric dams near Lake Baikal can cause the unique lake to dry out. It is shrinking
but the fate is not immense. It is not dried up.

Q4. Among the following cities, which one lies on a longitude closest to that of Delhi?

(a) Bengaluru
(b) Hyderabad
(c) Nagpur
(d) Pune

Answer: (a)

Option (a) is correct: The longitude of Delhi is 77.1025° E. Bengaluru with longitude 77.5946° E lies
on the longitude closest to that of Delhi.

Longitude of Hyderabad: 78.4867° E

Longitude of Nagpur: 79.0882° E

Longitude of Pune: 73.8567° E

Parallels of Latitude: The parallels of latitude refer to the angular distance, in degrees, minutes and
seconds of a point north or south of the Equator. Lines of latitude are often referred to as parallels.

Meridians of Longitude: The meridians of longitude refer to the angular distance, in degrees, minutes,
and seconds, of a point east or west of the Prime (Greenwich) Meridian. Lines of longitude are often
referred to as meridians.

Q5. With reference to the 'Global Alliance for Climate-Smart Agriculture (GACSA)', which of the
following statements is/are correct?

1. GACSA is an outcome of the Climate Summit held in Paris in 2015.


2. Membership of GACSA does not create any binding obligations.
3. India was instrumental in the creation of GACSA.

Select the correct answer using the code given below:

(a) 1 and 3 only


(b) 2 only
(c) 2 and 3 only
(d) 1, 2 and 3

Answer: (b)

Statement 1 is not correct: The Global Alliance for Climate-Smart Agriculture (GACSA) is an inclusive,
voluntary, and action-oriented multi-stakeholder platform on Climate-Smart Agriculture (CSA). GACSA
was launched in September 2014 in the margins of the UN Climate Summit. It was launched at COP
21 in Paris.
GACSA claims that it aspires to improve farmers’ agricultural productivity and incomes in a
sustainable way. It also hopes to build farmers’ resilience to extreme weather and changing climate
and reduce greenhouse gas emissions associated with agriculture.

Statement 2 is correct: Membership of GACSA does not create any binding obligations. Members
determine their particular voluntary actions according to their needs and priorities. Individuals cannot
be members, but they can join GACSA’s Action Groups.

Statement 3 is not correct. India was not instrumental in the creation of GACSA. Though, few Indian
NGOs are its members.

Climate-Smart Agriculture (CSA) is an approach to developing the technical, policy and investment
conditions to achieve sustainable agricultural development for food security under climate change.
The magnitude, immediacy, and broad scope of the effects of climate change on agricultural systems
create a compelling need to ensure comprehensive integration of these effects into national
agricultural planning, investments, and programs.

The concept of Climate-Smart Agriculture (CSA) was originally developed by FAO.

Climate-smart agriculture includes proven practical techniques by increasing the organic content of
the soil through conservation tillage, its water holding capacity increases, making yields more resilient
and reducing erosion, by promoting soil carbon capture also helps mitigate climate change. Another
example is integrated soil fertility management that can lower fertilizer costs, increase soil carbon and
improve yields. Climate-smart agriculture gives attention to landscape approaches, for example,
integrated planning of land, agriculture, forests, fisheries and water to ensure synergies are captured.

Q6. Consider the following pairs:

Towns sometimes mentioned in news - Country

1. Aleppo - Syria
2. Kirkuk - Yemen
3. Mosul - Palestine
4. Mazar-i-sharif - Afghanistan

Which of the pairs given above are correctly matched?

(a) 1 and 2 only


(b) 1 and 4 only
(c) 2 and 3 only
(d) 3 and 4 only

Answer: (b)

Pair 1 and pair 4 are correctly matched: Aleppo is in Syria. Mazar-i-sharif is located in Afghanistan.

Pair 2 and pair 4 are not correctly matched: Kirkuk and Mosul, both are in Iraq.

All these places appeared in news in context of war and violence. Terrorism and Ethic conflict are
the main reasons of violence. The ethnic dimension to social relations can be distinguished from
dimensions based on neighbourliness, national origin, race, and religion etc., but ethnic conflicts are
not a special class of conflicts. The significance of a shared ethnic origin varies with the location and
scale of social relations.

Q7. Consider the following statements:

1. The Earth's magnetic field has reversed every few hundred thousand years.
2. When the Earth was created more than 4000 million years ago, there was 54% oxygen and no
carbon dioxide.
3. When living organisms originated, they modified the early atmosphere of the Earth.

Which of the statements given above is/are correct?

(a) 1 only
(b) 2 and 3 only
(c) 1 and 3 only
(d) 1, 2 and 3

Answer: (c)

Statement 1 is correct: The Earth's magnetic field has flipped its polarity many times over the
millennia. Reversals are the rule, not the exception. Earth has settled in the last 20 million years into
a pattern of a pole reversal about every 200,000 to 300,000 years, although it has been more than
twice that long since the last reversal. A reversal happens over hundreds or thousands of years, and it
is not exactly a clean back flip. Magnetic fields morph and push and pull at one another, with multiple
poles emerging at odd latitudes throughout the process. Scientists estimate reversals have happened
at least hundreds of times over the past three billion years. And while reversals have happened more
frequently in "recent" years, when dinosaurs walked Earth, a reversal was more likely to happen only
about every few hundred thousand years. Earth's polarity is not constant.
Statement 2 is not correct: When the Earth was created more than 4000 million years ago, it was a
barren, rocky and hot object with a thin atmosphere of hydrogen and helium. Between 4,600 million
years and the present, led to the evolution of life on the surface of the planet. The early atmosphere
was probably mostly carbon dioxide, with little or no oxygen. There were smaller proportions of
water vapour, ammonia, and methane. As the Earth cooled down, most of the water vapour
condensed and formed the oceans.

Statement 3 is correct: There are three stages in the evolution of the present atmosphere. The first
stage is marked by the loss of primordial atmosphere. In the second stage, the hot interior of the earth
contributed to the evolution of the atmosphere. Finally, the composition of the atmosphere was
modified by the living world through the process of photosynthesis.

Q8. With reference to the circumstances in Indian agriculture, the concept of "Conservation
Agriculture" assumes significance. Which of the following fall under the Conservation Agriculture?

1. Avoiding the monoculture practices


2. Adopting minimum tillage
3. Avoiding the cultivation of plantation crops
4. Using crop residues to cover soil surface
5. Adopting spatial and temporal crop sequencing/crop rotations

Select the correct answer using the code given below:

(a) 1, 3 and 4 only


(b) 2, 3, 4 and 5 only
(c) 2, 4 and 5 only
(d) 1, 2, 3 and 5 only

Answer: (c)

Option (c) is correct: Conservation Agriculture is a set of soil management practices that minimize
the disruption of the soil's structure, composition, and natural biodiversity. Despite high variability
in the types of crops grown and specific management regimes, all forms of conservation agriculture
share three core principles, which are, adopting minimum tillage (just enough to get the seed into
the ground), using crop residues to cover soil surface, and adopting spatial and temporal crop
sequencing/crop rotations.
Conservation agriculture is largely the product of the collective efforts of a number of previous
agricultural movements, including no-till agriculture, agroforestry, green manures/cover crops, direct
planting/seeding, integrated pest management, and conservation tillage among many others.

Q9. The term "sixth mass extinction/sixth extinction" is often mentioned in the news in the context
of the discussion of

(a) Widespread monoculture practices in agriculture and large-scale commercial farming with
indiscriminate use of chemicals in many parts of the world that may result in the loss of good
native ecosystems.
(b) Fears of a possible collision of a meteorite with the Earth in the near future in the manner it
happened 65 million years ago that caused the mass extinction of many species including
those of dinosaurs.
(c) Large scale cultivation of genetically modified crops in many parts of the world and promoting
their cultivation in other parts of the world which may cause the disappearance of good native
crop plants and the loss of food biodiversity.
(d) Mankind's overexploitation/misuse of natural resources, fragmentation/loss of natural
habitats, destruction of ecosystems, pollution and global climate change.

Answer: (d)

Mass extinction refers to a substantial increase in the degree of extinction or when the Earth loses
more than three-quarters of its species in a geologically short period of time. So far, during the entire
history of the Earth, there have been five mass extinctions. The sixth, which is ongoing, is referred to
as the Anthropocene extinction.

The ‘Sixth Extinction’ is presently in progress, different from the previous episodes. The difference is
in the rates; the current species extinction rates are estimated to be 100 to 1,000 times faster than in
the pre-human times and our activities are responsible for the faster rates. Ecologists warn that if
the present trends continue, nearly half of all the species on earth might be wiped out within the next
100 years.

Humanity needs a relatively stable climate, flows of fresh water, agricultural pest and disease-vector
control and pollination for crops, all services that will be impacted as the sixth mass extinction
accelerates.
Option (d) is correct: The term "sixth mass extinction/sixth extinction" is in the context of
overexploitation/misuse of natural resources, fragmentation/loss of natural habitats, destruction of
ecosystems, pollution and global climate change.

Q10. Which of the following statements best describes "carbon fertilization"?

(a) Increased plant growth due to increased concentration of carbon dioxide in the atmosphere
(b) Increased temperature of Earth due to increased concentration of carbon dioxide in the
atmosphere
(c) Increased acidity of oceans as a result of increased concentration of carbon dioxide in the
atmosphere
(d) Adaptation of all living beings on Earth to the climate change brought about by the increased
concentration of carbon dioxide in the atmosphere

Answer: (a)

Option (a) is correct: A higher concentration of carbon dioxide in the atmosphere aids
photosynthesis, which in turn contributes to increased plant growth. This is known as the carbon
fertilization effect.

Under higher CO2, plants do not lose so much water through their leaves, thereby less impacted by
drier conditions. Carbon dioxide fertilization is not the only cause of increased plant growth- nitrogen,
land cover change and climate change by way of global temperature, precipitation and sunlight
changes all contribute to the greening effect.

Q11. Which one of the following is an artificial lake?

(a) Kodaikanal (Tamil Nadu)


(b) Kolleru (Andhra Pradesh)
(c) Nainital (Uttarakhand)
(d) Renuka (Himachal Pradesh)

Answer: (a)

Option (a) is correct: Kodaikanal Lake, which is one of the finest man-made (artificial) lakes in India,
is located in the state of Tamil Nadu.

Kolleru lake, one of the largest freshwater lakes in India, (it was designated a sanctuary in October
1999) is situated between the Krishna and West Godavari districts of Andhra Pradesh. Kolleru is one
of the most important wetlands of India. The atapaka Bird Sanctuary, part of the Kolleru Lake, has
been identified as the world’s largest home for the spot-billed pelican. It is a Ramsar site.
Naini Lake is the sole source of drinking water for Nainital town, an important tourist destination in
Uttarakhand state. It is a natural lake.

Renuka lake is situated in the Sirmour district of Himachal Pradesh in India. One of the popular
temples of Himachal Pradesh, Renukaji is situated on the bank of this lake. It is a Ramsar site.

Q12. Which of the following is/are the possible consequence/s of heavy sand mining in riverbeds?

1. Decreased salinity in the river


2. Pollution of groundwater
3. Lowering of the water - table

Select the correct answer using the code given below:

(a) 1 only
(b) 2 and 3 only
(c) 1 and 3 only
(d) 1, 2 and 3

Answer: (b)

“Sand mining” is a practice that is used to extract sand from various environments, such as beaches,
inland dunes and dredged from ocean beds, and riverbeds of deltaic regions. The mining is in
operation in all the continents of the Globe. Environmental problems occur when the rate of
extraction of sand, gravel and other materials exceeds the rate of deposition.

Option (b) is correct: Sand is vital for sustenance of rivers. It has several impacts on the river
environment as:

● It disturbs and completely removes the habitat from the mined zones. Moreover, it leads to
changes in its channel form, physical habitats, and food webs – the river’s ecosystem.
● It also increases the velocity of flow in rivers which destroy flow-regime and eventually erodes
the riverbanks.
● The sediment in the mining site, if it contains toxic material due to long time accumulation,
will cause groundwater pollution.
● Riverbed becomes dry due to exposure to solar radiation, decreasing the surface and
groundwater. It causes lowering of the water tables.
● Depletion of sand in the streambed causes the deepening of rivers and estuaries, and the
enlargement of river mouths and coastal inlets. It leads to decreased salinity in the river.
Q13. With reference to agricultural soils, consider the following statements:

1. A high content of organic matter in soil drastically reduces its water holding capacity.
2. Soil does not play any role in the sulphur cycle.
3. Irrigation over a period of time can contribute to the salinization of some agricultural lands.

Which of the statements given above is/are correct?

(a) 1 and 2 only


(b) 3 only
(c) 1 and 3 only
(d) 1, 2 and 3

Answer: (b)

Organic matter affects both the chemical and physical properties of the soil and its overall health.
Properties influenced by organic matter include soil structure; moisture holding capacity; diversity
and activity of soil organisms, both those that are beneficial and harmful to crop production; and
nutrient availability.

Statement 1 is not correct: Organic matter contributes to the stability of soil aggregates and pores
through the bonding or adhesion properties of organic materials, such as bacterial waste products,
organic gels, fungal hyphae and worm secretions and casts. Moreover, organic matter intimately
mixed with mineral soil materials has a considerable influence in increasing water holding capacity.

Especially in the topsoil, where the organic matter content is greater, more water can be stored.

Statement 2 is not correct: Sulphur is one of three nutrients that are cycled between the soil, plant
matter and the atmosphere. The Sulphur cycle describes the movement of Sulphur through the
atmosphere, mineral and organic forms, and through living things. Although Sulphur is primarily found
in sedimentary rocks, it is particularly important to living things because it is a component of many
proteins. Hence, soil plays a role in the sulphur cycle.
Statement 3 is correct: A fairly large area of arable land in the irrigated zones of India is becoming
saline because of overirrigation. The salt lodged in the lower profiles of the soil comes up to the
surface and destroys its fertility. Chemical fertilisers in the absence of organic manures are also
harmful to the soil. Unless the soil gets enough humus, chemicals harden it and reduce its fertility in
the long run. This problem is common in all the command areas of the river valley projects, which
were the first beneficiaries of the Green Revolution. According to estimates, about half of the total
land of India is under some degree of degradation.

Q14. The Partnership for Action on Green Economy (PAGE), a UN mechanism to assist countries
transition towards greener and more inclusive economies, emerged at

(a) The Earth Summit on Sustainable Development 2002, Johannesburg


(b) The United Nations Conference on Sustainable Development 2012, Rio de Janeiro
(c) The United Nations Framework Convention on Climate change 2015, Paris
(d) The World Sustainable Development Summit 2016, New Delhi

Answer: (b)

Option (b) is correct: The Partnership for Action on Green Economy (PAGE), a United Nation
mechanism to assist countries transition towards greener and more inclusive economies, emerged at
The United Nations Conference on Sustainable Development 2012, Rio de Janeiro.
PAGE seeks to put sustainability at the heart of economic policies and practices to advance the 2030
Agenda for Sustainable Development and supports nations and regions in reframing economic
policies and practices around sustainability to foster economic growth, create income and jobs,
reduce poverty and inequality, and strengthen the ecological foundations of their economies.

PAGE brings together five UN agencies which are the UN Environment, International Labour
Organization, UN Development Programme, UN Industrial Development Organization, and UN
Institute for Training and Research.

Q15. Consider the following statements:

1. The Barren Island volcano is an active volcano located in the Indian territory.
2. Barren Island lies about 140 km east of Great Nicobar.
3. The last time the Barren Island volcano erupted was in 1991 and it has remained inactive since
then.

Which of the statements given above is/are correct?

(a) 1 only
(b) 2 and 3
(c) 3 only
(d) 1 and 3

Answer: (a)

Statement 1 is correct: Andaman and Nicobar Islands are separated by a water body which is called
the Ten-degree channel. It is believed that these islands are an elevated portion of submarine
mountains. However, some smaller islands are volcanic in origin. Barren island, the only active
volcano in India is also situated in the Nicobar Islands.

Statement 2 is not correct: The Barren Island lies about 140 Km from Port Blair.

Statement 3 is not correct: The last time the Barren Island volcano erupted was in 2018. It was a
stratovolcano composed of lava, rock fragments, and volcanic ash. On the west side of the island is a
caldera formed by an explosive eruption in the Pleistocene era.
Q16. Why is a plant called Prosopis juliflora often mentioned in news?

(a) Its extract is widely used in cosmetics.


(b) It tends to reduce the biodiversity in the area in which it grows.
(c) Its extract is used in the synthesis of pesticides.
(d) None of the aboves

Answer: (b)

Option (b) is correct: Prosopis juliflora (vilayati kikar or vilayati babul), an exotic tree, is one of the
top invaders in India. It was introduced into India from South America and which has become
naturalised all-over north India. This tree is also affecting the nesting success of birds. It tends to
reduce the biodiversity in the area in which it grows.

Invasive plants are those which-spread to regions other than their native and thrive to the extent of
becoming a threat to local species. Alien invasive species occur in all groups of plants and animals.
They include competitors, predators, pathogens, and parasites. They have invaded almost every
type of native ecosystem and have caused hundreds of extinctions.

Q17. Consider the following statements:

1. Most of the world's coral reefs are in tropical waters.


2. More than one-third of the world's coral reefs are located in the territories of Australia,
Indonesia and Philippines.
3. Coral reefs host far more number of animal phyla than those hosted by tropical rainforests.

Which of the statements given above is/are correct?

(a) 1 and 2 only


(b) 3 only
(c) 1 and 3 only
(d) 1, 2 and 3

Answer: (d)

Coral Reefs are most commonly known as the rainforest of oceans. They are the underwater
structures that are formed of coral polyps, held together by calcium carbonate.

Coral polyps are short-lived microscopic organisms, which live in colonies. They flourish in shallow,
mud free and warm waters. They secrete calcium carbonate. The coral secretion and their skeletons
from coral deposits, in the form of reefs, are mainly of three kinds: barrier reef, fringing reef and
atolls. The Great Barrier Reef of Australia is a good example of the first kind of coral reefs.

Statement 1 is correct: Mostly the coral reefs are in tropical waters, located between the Tropics of
Cancer and Capricorn, in the Pacific Ocean, the Indian Ocean, the Caribbean Sea, the Red Sea, and the
Persian Gulf.

Statement 2 is correct: Indonesia, followed by Australia and the Philippines are the largest reef
nations, while France comes in fourth. More than one-third of the world's coral reefs are located in
the territories of Australia, Indonesia, and Philippines.

Statement 3 is correct: Coral reefs are the most diverse of all marine ecosystems. Reefs cover just a
tiny fraction (less than one percent) of the earth’s surface and less than two percent of the ocean
bottom. Because they are so diverse, coral reefs are often called the rainforests of the sea. Coral reefs
host a greater number of animal phyla than those hosted by tropical rainforests.

Q18. "Momentum for Change: Climate Neutral Now" is an initiative launched by

(a) The Intergovernmental Panel on Climate Change


(b) The UNEP Secretariat
(c) The UNFCCC Secretariat
(d) The World Meteorological Organization
Answer: (c)

Option (c) is correct: "Momentum for Change: Climate Neutral Now" is an initiative launched by the
UNFCCC Secretariat.

The UNFCCC secretariat launched its Climate Neutral Now initiative in 2015. The following year, the
secretariat launched a new pillar under its Momentum for Change initiative focused on Climate
Neutral Now, as part of larger efforts to showcase successful climate action around the world.

Climate neutrality is a three-step process, which requires individuals, companies, and governments
to measure their climate footprint, reduce their emissions as much as possible, and offset what they
cannot reduce with UN certified emission reductions.

Q19. Consider the following pairs:

Regions sometimes mentioned in news – Country

1. Catalonia - Spain
2. Crimea - Hungary
3. Mindanao - Philippines
4. Oromia - Nigeria

Which of the pairs given above are correctly matched?

(a) 1, 2 and 3
(b) 3 and 4 only
(c) 1 and 3 only
(d) 2 and 4 only

Answer: (c)

Pair 1 is correctly matched: Catalonia is an autonomous community in the north-eastern corner of


Spain. It is one of Spain’s wealthiest regions. It consists of four provinces, such as Barcelona, Girona,
Lleida, and Tarragona.

Pair 2 is not correctly matched: Crimea is a peninsula located on the northern coast of the Black Sea.
In 2014, Russia seized Crimea from Ukraine in an illegal move that violated the territorial integrity of
the former Soviet republic and sparked a war that has displaced nearly 2 million people and destroyed
the country’s infrastructure.
Pair 3 is correctly matched: Mindanao is the second-largest island in the Philippines, after Luzon.

Pair 4 is not correctly matched: The Oromo people are the native inhabitants of Ethiopia, Eastern
Africa. Their population is estimated to be 55 million, which makes it the largest ethnic group in
Eastern Africa.

Q20. In which one of the following States is Pakhui Wildlife Sanctuary located?

(a) Arunachal Pradesh


(b) Manipur
(c) Meghalaya
(d) Nagaland

Answer: (a)

Option (a) is correct: Pakhui Wildlife Sanctuary lies in the foothills of the Eastern Himalaya, in the
East Kameng District of Arunachal Pradesh. It was declared a Tiger Reserve in 2002. The habitat
comprises lowland semi-evergreen, evergreen, and Eastern Himalayan broadleaf forests.

National Parks in Arunachal Pradesh: Mouling and Namdapha, etc.

Wildlife Sanctuary in Arunachal Pradesh: D' Ering Memorial, Dibang, Eagle Nest, Itanagar, Kane,
Mehao, Pakhui, Sessa Orchid, etc.

Q21. Consider the following statements:

1. The definition of "Critical Wildlife Habitat" is incorporated in the Forest Rights Act, 2006.
2. For the first time in India, Baigas have been given Habitat Rights.
3. Union Ministry of Environment, Forest and Climate Change officially decides and declares
Habitat Rights for Primitive and Vulnerable Tribal Groups in any part of India.

Which of the statements given above is/are correct?

(a) 1 and 2 only


(b) 2 and 3 only
(c) 3 only
(d) 1, 2 and 3

Answer: (a)
Statement 1 is correct: The definition ‘Critical Wildlife Habitat’ is incorporated in the Scheduled
Tribes and Other Traditional Forest Dwellers (Recognition of Forest Rights) Act, 2006.
Statement 2 is correct: Baiga Tribals became India’s first community to get Habitat Rights. Baigas
are considered as a Particularly Vulnerable Tribal Group (PVTG) in the Indian Constitution and rely
mostly on shifting cultivation, forest produce and fishing for sustenance. The tribe numbers only
150,000 people spread over forested areas of Madhya Pradesh and Chhattisgarh.
Statement 3 is not correct: The definition of the Habitat Rights was incorporated through an
Amendment in the FRA in 2012. As per the amendment, the District Level Committee under the Act
shall ensure that all PVTGs receive Habitat Rights, in the Consultation with the concerned traditional
institutions of these groups, after filing claims before the Gram Sabha.

Q22. With reference to organic farming in India, consider the following statements:

1. 'The National Programme for Organic Production' (NPOP) is operated under the guidelines
and directions of the Union Ministry of Rural Development.
2. 'The Agricultural and Processed Food Products Export Development Authority' (APEDA)
functions as the Secretariat for the implementation of NPOP.
3. Sikkim has become India's first fully organic State.

Which of the statements given above is/are correct?

(a) 1 and 2 only


(b) 2 and 3 only
(c) 3 only
(d) 1, 2 and 3

Answer: (b)

Statement 1 is not correct: The Ministry of Commerce and Industry has implemented the National
Programme for Organic Production (NPOP) since 2001.

The objectives of NPOP are to provide the means of evaluation of certification programmes for
organic agriculture and products (including wild harvest, aquaculture, livestock products) as per the
approved criteria, to accredit certification programmes of Certification Bodies seeking accreditation,
to facilitate certification of organic products in conformity with the prescribed standards, to facilitate
certification of organic products in conformity with the importing countries organic standards as per
equivalence agreement between the two countries or as per importing country requirements, to
encourage the development of organic farming and organic processing.

Statement 2 is correct: The Agricultural and Processed Food Products Export Development Authority
(APEDA) shall function as the Secretariat for the implementation of the NPOP. APEDA was
established by the Government of India under the Agricultural and Processed Food Products Export
Development Authority Act passed by the Parliament in December 1985.

Statement 3 is correct: Sikkim has become India’s first fully organic state by implementing organic
practices on around 75,000 hectares of agricultural land.

Q23. Consider the following:

1. Birds
2. Dust blowing
3. Rain
4. Wind blowing

Which of the above spread plant diseases?

(a) 1 and 3 only


(b) 3 and 4 only
(c) 1, 2 and 4 only
(d) 1, 2, 3 and 4

Answer: (d)

Plant diseases are caused by a pathogenic organism such as a fungus, bacterium, mycoplasma, virus,
viroid, nematode, or parasitic flowering plant. Plant diseases can be classified as endemic diseases
(prevalent in and confined to a particular locality), epidemic diseases (occur incidentally and
occasionally in a particular locality) and sporadic diseases (occur at very irregular intervals and
locations).

Option (d) is correct: Plant diseases spread through direct transmission (the pathogen is carried
externally or internally on the seed or planting material like cuttings, sets, tubers, bulbs etc.) or
through indirect transmission (the pathogen spreads itself by way of its persistent growth or certain
structures of the pathogen carried independently by natural agencies like wind, water (rain), animals,
insects, mites, nematodes, birds etc.
History and Art & Culture

Q1. Consider the following events:

1. The first democratically elected communist party government formed in a State in India.
2. India's then largest bank, 'Imperial Bank of India', was renamed 'State Bank of India'.
3. Air India was nationalised and became the national carrier.
4. Goa became a part of independent India.

Which of the following is the correct chronological sequence of the above events?

(a) 4 - 1 - 2 - 3
(b) 3 - 2 - 1 - 4
(c) 4 - 2 - 1 - 3
(d) 3 - 1 - 2 - 4

Answer: (b)

Option (b) is correct: In 1953, India nationalized all Indian airlines, creating two corporations—one
for domestic service, called Indian Airlines Corporation (merging Air-India Limited with six lesser
lines), and one for international service, Air-India International Corporation.

The Reserve Bank of India, which is the Central Bank of India, acquired a controlling interest in the
Imperial Bank of India in 1955. On 1 July 1955, the Imperial Bank of India became the State Bank of
India.

The Kerala Legislative Assembly election of 1957 was the first Assembly election in the Indian state
of Kerala. Indian state of Kerala became the first Communist Party barring the PCI in San Marino to
win elections in a parliamentary democracy. The Communist Party formed the government.

On 19 December 1961, Goa was liberated from the Portuguese rule and became a part of independent
India. After conferring statehood on Goa on 30 May 1987, Daman and Diu was made a separate Union
Territory.

Q2. Which of the following led to the introduction of English Education in India?

1. Charter Act of 1813


2. General Committee of Public Instruction, 1823
3. Orientalist and Anglicist Controversy

Select the correct answer using the code given below:

(a) 1 and 2 only

(b) 2 only

(c) 1 and 3 only

(d) 1, 2 and 3

Answer: (d)

Option (d) is correct: The introduction of English Education in India was first put forward by
the Charter Act of 1813 and Orientalist and Anglicist Controversy. Then in 1823, the Governor-
General-in Council appointed a “General Committee of Public Instruction”, which had the
responsibility to grant one lakh of rupees for education. That Committee consisted of ten European
members of which Lord Macaulay was the President.

Q3. In 1920, which of the following changed its name to "Swarajya Sabha"?

(a) All India Home Rule League


(b) Hindu Mahasabha
(c) South Indian Liberal Federation
(d) The Servants of India Society

Answer: (a)

Option (a) is correct: The year 1919 opened with two main political bodies in the field, viz., the All
India Home Rule League established in 1916 mainly by Mrs. Besant and Tilak's Indian Home Rule
League started in 1917. The Home Rule League demanded self-government based on the Irish
model. Tilak's League with its stronghold in the Deccan. Mrs. Besant on the other hand had for some
time been losing ground everywhere. All India Home Rule League which in October 1920 was
renamed the "Swarajya Sabha” with the avowed object of securing complete Swaraj for India in
accordance with the wishes of the people of India.

Q4. Which among the following events happened earliest?


(a) Swami Dayanand established Arya Samaj.
(b) Dinabandhu Mitra wrote Neel Darpan.
(c) Bankim Chandra Chattopadhyay wrote Anandmath.
(d) Satyendranath Tagore became the first Indian to succeed in the Indian Civil Services
Examination.

Answer: (b)

Option (b) is correct: Neel Darpan is a famous Bengali play which was written by Dinabandhu Mitra
in 1858-59, published in Dhaka in 1860. The aim of this play was to protest against the imperial rule
of British rule in India. The main context of the play is on the event of Indigo Revolt in Bengal.

Swami Dayanand established the Arya Samaj in April 1875, in Bombay. Formed with the motto
Krinvanto Vishvam Aryam (Make this world noble), the socio-cultural movement aimed to reform
society by raising awareness about Vedic knowledge among people. The movement believed in
infallibility of Vedas and took them as the only truth and source of all the knowledge. It also believed
that post-Vedic texts such as Puranas were responsible for polluting the Vedic religion. It opposed the
idol-worship and reincarnation theory of God.

Bankim Chandra Chattopadhyay wrote Anandamath in 1882. It was set in the background of the
Sanyasi Bidroho (rebellion of monks in late 18th century) and is one of key works on Bengal’s
nationalism.

Satyendranath Tagore was selected for the Indian Civil Service in June 1863. He completed his
probationary training and returned to India in November 1864.

Q5. With reference to educational institutions during colonial rule in India, consider the following
pairs:

Institution Founder

1. Sanskrit College at Benaras William Jones


2. Calcutta Madarsa Warren Hastings
3. Fort William College Arthur Wellesley

Which of the pairs given above is/are correct?


(a) 1 and 2 only

(b) 2 only

(c) 1 and 3 only

(d) 3 only

Answer: (b)

Pair 1 is not correctly matched: The first generation of British administrators in India like Warren
Hastings, William Jones and Jonathan Duncan popularised the view that India had a glorious past
which had subsequently degenerated. These scholars and administrators were called Orientalists.
They were keen to learn and propagate Indian languages and traditions. This, they thought, would
ensure a better understanding of India which would eventually strengthen their rule over this country.
Hence, in 1794, Jonathan Duncan founded the Sanskrit College at Benares.

Pair 2 is correctly matched: Warren Hastings set up Calcutta Madrasa in 1781 for the study and
learning of Persian and Arabic. This was done as the British hoped to win a place in the hearts of the
“natives”; only then could the alien rulers expect to be respected by their subjects.

Pair 3 is not correctly matched: There was a strong urge to make local British administrators familiar
with Indian culture and tradition. Fort William College founded by Richard Wellesley in 1801 to train
the young British recruits to the civil service in India was meant to serve primarily this purpose. This
college became an important centre for producing knowledge on and about India. It had many
departments devoted exclusively to research on Indian languages and literature.

Q6. The staple commodities of export by the English East India Company from Bengal in the middle
of the 18th century were:

(a) Raw cotton, oil-seeds and opium


(b) Sugar, salt, zinc and lead
(c) Copper, silver, gold, spices and tea
(d) Cotton, silk, saltpetre and opium

Answer: (d)

Option (d) is correct: Crucial to the emergence of a powerful capitalist economy, British-Indian
territory was developed as a source of food and raw material for Britain. In the 1750s, fine cotton
and silk was exported from India to markets in Europe, Asia, and Africa. Also, raw materials which
chiefly consisted of saltpetre, opium and indigo, accounted for most of India's exports.

Q7. Which one of the following is a very significant aspect of the Champaran Satyagraha?

(a) Active all-India participation of lawyers, students and women in the National Movement
(b) Active involvement of Dalit and Tribal communities of India in the National Movement
(c) Joining of peasant unrest to India's National Movement
(d) Drastic decrease in the cultivation of plantation crops and commercial crops

Answer: (c)

Option (c) is correct: The Champaran Satyagraha of 1917 was India’s first Civil Disobedience
Movement that opened a new phase in the national movement by joining it to the great struggle of
the Indian peasantry for bread and land. Gandhi led the struggle of indigo workers as India’s first
Satyagraha that set the pace for involvement of peasant unrest to the National Movement.

The Montagu-Chelmsford Reforms introduced Dyarchy in the provinces with the Government of
Indian Act, 1919. The Moderates welcomed these reforms, while the Extremists rejected them. The
Rowlatt Act was also passed in 1919 to suppress political violence. At this juncture appeared a new
face in the political arena of India’s freedom movement. This was Gandhi, who filled the vacuum
created in the top leadership of the Congress. Gandhi had led the movement against discrimination
meted out to Indians in South Africa. He had used the political weapon called Satyagraha (Truth
force, or Love force, or Soul force). His first triumph in India was the Champaran Satyagraha.

Q8. Who among the following were the founders of the "Hind Mazdoor Sabha" established in
1948?

(a) B. Krishna Pillai, E.M.S. Namboodiripad and K.C. George


(b) Jayaprakash Narayan, Deen Dayal Upadhyay and M.N. Roy
(c) C.P. Ramaswamy Iyer, K. Kamaraj and Veeresalingam Pantulu
(d) Ashok Mehta, T.S. Ramanujam and G.G. Mehta

Answer: (d)

Option (d) is correct: The Hind Mazdoor Sabha (HMS) was founded in Howrah in West Bengal on 29
December 1948, by socialists, Forward Bloc followers and independent unionists. It was founded by,
Ashok Mehta, T.S. Ramanujam, G.G. Mehta, and other members. R.S. Ruikar was elected as the
President and Ashok Mehta as the General Secretary.

Q9. Which one of the following statements does not apply to the system of Subsidiary Alliance
introduced by Lord Wellesley?

(a) To maintain a large standing army at others expense


(b) To keep India safe from Napoleonic danger
(c) To secure a fixed income for the Company
(d) To establish British paramountcy over the Indian States

Answer: (c)

The doctrine of Subsidiary Alliance was introduced by Lord Wellesley, British Governor-
General in India from 1798 to 1805. Under the subsidiary alliance system, the ruler of the allying
Indian State was obliged to accept the permanent stationing of a British force within his territory and
to pay a subsidy for its maintenance.

Option (c) is correct: Some Important features of Subsidiary Alliance are, to maintain a large
standing army at others expense, to keep India safe from Napoleonic danger, to establish British
paramountcy over the Indian States, etc. To secure a fixed income for the Company was not a part
of it.

Q10. After the Santhal Uprising subsided, what was/were the measure/measures taken by the
colonial government?

1. The territories called ‘Santhal Parganas’ were created.


2. It became illegal for a Santhal to transfer land to a non-Santhal.

Select the correct answer using the code given below:

(a) 1 only

(b) 2 only

(c) Both 1 and 2

(d) Neither 1 nor 2


Answer: (c)

Santhals are a group of tribes concentrated in the state of Jharkhand. The Santhal Revolt took place in
1855-56. This was the first peasant movement which took place in India. The revolt has reference to
the establishment of the permanent land settlement of 1793.

Statement 1 is correct: Santhal rebellion was led by four Murmu brothers named Sindhu, Kanhu,
Chand and Bhairav against the oppressive zamindari system. The Santhals showed exemplary courage
in fighting against the British, despite being beaten and harassed. In order to be able to rule the area,
the British finally agreed on demands of Santhals following which the district of Santhal Pargana was
created in 1885, carving out 5,500 square miles from the districts of Bhagalpur and Birbhum.

Statement 2 is correct: Also, the British government enacted laws to ensure that the tribal land is not
taken away by outsiders (dikus). This means, it became illegal for a Santhal to transfer land to a non-
Santhal.

Q11. Economically one of the results of the British rule in India in the 19th century was the

(a) increase in the export of Indian handicrafts


(b) growth in the number of Indian owned factories
(c) commercialization of Indian agriculture
(d) rapid increase in the urban population

Answer: (c)

Option (c) is correct: Commercialization of Indian agriculture was the result of British rule in India in
the 19th century. Farmers were forced to grow Indigo and other cash crops which were exported to
Great Britain as raw materials of the English factories. In the 19th century, agriculture became
commercial due to the use of modern agricultural equipment and Indians started producing crops for
commercial use and on a large scale basis. Commercialisation of agriculture further enhanced the
speed of transfer of ownership of land thereby increasing the number of landless laborers.

It also brought in many merchants, traders and middlemen who further exploited the situation; There
was an enormous drain of wealth from India to Britain due to the various economic policies.
Additional financial burden was placed on India due to expenditures on salaries, pensions, and training
of military and civilian staff employed by the British to rule India.

Q12. Regarding Wood's Dispatch, which of the following statements are true?
1. Grants-in-Aid system was introduced.
2. Establishment of universities was recommended.
3. English as a medium of instruction at all levels of education was recommended.

Select the correct answer using the code given below:

(a) 1 and 2 only


(b) 2 and 3 only
(c) 1 and 3 only
(d) 1, 2 and 3

Answer: (a)

Charles Wood was a British Liberal politician and Member of Parliament. He served as Chancellor of
the Exchequer from 1846 to 1852. He later became the President of the Board of Control of the East
India Company. In 1854, the “Wood’s despatch” was sent to the Governor General Lord Dalhousie by
him. This document is considered as the "Magna Carta of English Education in India".

Statement 1 is correct: As per this Wood's Despatch an education department was to be set up in
every province along with the provision of grants in aid to the affiliated private schools.

Statement 2 is correct: The Wood's Despatch also recommended the establishment of three
universities in the Presidency towns of Calcutta, Bombay and Madras on the model of the London
University.

Statement 3 is not correct: It also wanted to promote the study of English as well as Indian regional
languages and classical languages like Arabic, Persian and Sanskrit. It did not recommend English as
a medium of instruction at all levels of education, only for higher studies it was recommended.

Q13. He wrote biographies of Mazzini, Garibaldi, Shivaji and Shrikrishna, stayed in America for some
time; and was also elected to the Central Assembly. He was

(a) Aurobindo Ghosh


(b) Bipin Chandra Pal
(c) Lala Lajpat Rai
(d) Motilal Nehru

Answer: (c)
Option (c) is correct: Lajpat Rai was a famous nationalist in India who was also known for
his proficiency in writing. To rouse and inspire the Punjabis with patriotic zeal, he wrote the
biographies of Mazzini, Garibaldi, Shivaji, Dayanand and Shri Krishna, besides other important
works in English, Hindi, and Urdu. He was famously called the “Lion of Punjab”.

Q14. Which one of the following foreign travellers elaborately discussed about diamonds and
diamond mines of India?

(a) Francois Bernier

(b) Jean-Baptiste Tavernier

(c) Jean de Thevenot

(d) Abbe Barthelemy Carre

Answer: (b)

Option (b) is correct: Jean-Baptiste Tavernier (1605‒89) was one of the most renowned travelers of
17th century Europe. The son of a French Protestant who had fled Antwerp to escape religious
persecution, Tavernier was a jewel merchant who between 1632 and 1668 made six voyages to the
East. The countries he visited (most more than once) included present-day Cyprus, Malta, Turkey,
Syria, Iraq, Iran, Afghanistan, Pakistan, India, Sri Lanka, and Indonesia. While on his travels to India
he discovered and discussed diamonds and diamond mines of India extensively.

Q15.Consider the following pairs:


Craft Heritage of

1. Puthukkuli shawls Tamil Nadu

2. Sujni embroidery Maharashtra

3. Uppada Jamdani saris Karnataka

Which of the pairs given above is/are correct?

(a) 1 only
(b) 1 and 2 only
(c) 3 only
(d) 2 and 3 only

Answer: (a)
Pair 1 is correctly matched: Puthukkuli shawls are made by Todas of Nilgiri hills in Tamil Nadu. It is
locally called Pugur, meaning flower. The fine and intricate Toda embroidery is done by tribal men and
women on shawls.

Pair 1 and pair 2 are not correctly matched: Sujini and Khatwa Embroidery is famous as well as
beautiful work of handicraft in Bihar. Jamdani style of weaving originated in Bangladesh. Uppada
Sari is a sari style woven in Uppada of East Godavari district in Andhra Pradesh. Jamdani is a
‘cotton’ fibre woven delicately into the finest muslin. Uppada saree was registered by the
Geographical Indications Registry tag (GI Tag) for the jamdani technique in 2009. In 2013, the
traditional art of weaving jamdani was declared a UNESCO Intangible Cultural Heritage of Humanity.

Q16. With reference to cultural history of India, consider the following statements:

1. Most of the Tyagaraja Kirtis are devotional songs in Praise of Lord Krishna.

2. Tyagaraja created several new ragas.

3. Annamacharay and Tyagaraja are contemporaries.

4. Annamacharaya kirtanas are devotional songs in praise of Lord Venkateshwara.

Which of the statements given above are correct?

(a) 1 and 3 only


(b) 2 and 4 only
(c) 1, 2 and 3
(d) 2, 3 and 4

Answer: (b)

Statement 1 is not correct: Saint Tyagaraja was a renowned composer of Carnatic music. He
composed thousands of devotional compositions, mostly in Telugu in the praise of Lord Rama, many
of which remain popular even till this date. He saw the reigns of four kings of Maratha dynasty —
Tulaja II (1763-1787), Amarasimha (1787-1798), Serfoji II (1798-1832) and Sivaji II (1832-1855), but he
served none of them.

Statement 2 is correct: Tyagaraja introduced and gave life to several unusual new ragas in Carnatic
music. Some he created while others he found as mere scales in books and infused musicality into
them.
Statement 3 is not correct: Sri Tallapaka Annamacharya was a saint composer in the 15th century, who
hailed from the Tallapaka village, in the Rajampet mandal of Cuddapah district, Andhra Pradesh, India.
While Tyagaraja belonged to the 18th century. Hence, they were not contemporaries.

Statement 4 is correct.: Annamacharaya was the earliest known Indian musician to compose songs
called Sankirtanas in praise of god Venkateshwara, a form of Vishnu.

Q17. The well-known painting “Bani Thani” belongs to the


(a) Bundi School
(b) Jaipur School
(c) Kangra School
(d) Kishangarh School

Answer: (d)

Option (d) is correct: The Kishangarh province in Rajasthan is famous for its Bani Thani paintings. It
is a style with highly exaggerated features like long necks, large, almond shaped eyes, and long fingers.
This style of painting essentially depicts Radha and Krishna as divine lovers, and beautifully portrays
their mystical love.
Kishangarh miniature painting reached a peak in the eighteenth century, during the rule of Raja
Sawant Singh, who fell in love with a slave girl, Bani Thani and commanded his artists to portray
himself and her as Krishna and Radha. Other themes of Bani Thani paintings include portraits, court
scenes, dancing, hunting, music parties, nauka vihar (lovers travelling in a boat), Krishna
Lila, Bhagavata Purana and various festivals like Holi, Diwali, Durga puja, and Dussehra.

Q18. Consider the following pairs:


Tradition State

1. Chapchar Kut festival Mizoram

2. Khongiom Parba Ballad Manipur

3. Thang-Ta dance Sikkim

Which of the pairs given above is/are correct?

(a) 1 only
(b) 1 and 2 only
(c) 3 only
(d) 2 and 3 only
Answer: (b)

Pair 1 is correctly matched: Chapchar Kut, a popular spring festival, is celebrated by the Mizo people
of Mizoram. Celebrated in early March, every year, it is one of the 3 festivals that marks the various
stages of the agricultural cycle. The other two are Mim Kut and Pawl Kut. This festival celebrates the
completion of the jhum operations in the state.
Pair 2 is correctly matched: Khongjom Parba ballad is a style of ballad singing from Manipur. It is
accompanied with the sounds of a dholak or drum. This art form depicts stories of the heroic battle
of Khonjom, fought by the people of Manipur against the British empire in April 1891. It is one of the
most popular musical art forms of the state and spurs the spirit of patriotism and nationalism in the
audience.

Pair 3 is not correctly matched: Thang-Ta is a Manipuri art form. It combines ritual, demonstration
and combat and involves a variety of dance forms and warrior drills.

Q19. With reference to the religious practices in India, the “Sthanakvasi” sect belongs to
(a) Buddhism
(b) Jainism
(c) Vaishnavism
(d) Shaivism

Answer: (b)

Option (b) is correct: Sthanakvasi is a sect of Svetambara Jainism founded by a merchant named
Lavaji in 1653 AD. It believes that idol worship is not essential in the path of soul purification and
attainment of Nirvana or Moksha. The sect is essentially a reformation of the one founded on
teachings of Lonka, a fifteenth-century Jain reformer.

Jainism is an ancient religion that follows the philosophy of liberation and also a path to spiritual purity
and enlightenment through disciplined nonviolence to all living creatures. Jainism came into
prominence in 6th century B.C. when Lord Mahavira propagated the religion. There were 24 great
teachers, the last of whom was Lord Mahavira. Jain order has been divided into two major sects:
Digambar and Svetambara.

Q20. With reference to the cultural history of India, consider the following statements:

1. White marble was used in making Buland Darwaza and Khankah at Fatehpur Sikri.

2. Red sandstone and marble were used in making Bara Imambara and Rumi Darwaza at
Lucknow.
Which of the statements given above is/are correct?

(a) 1 only
(b) 2 only
(c) Both 1and 2
(d) Neither 1 nor 2

Answer: (a)

Statement 1 is correct: The Buland Darwaza is made of red and buff sandstone, decorated by white
and black marble and is higher than the courtyard of the mosque. The construction of the Buland
Darwaza was inspired by Timurid architecture. Along with Humayun’s Tomb, its monumentality
reflects its Central Asian origins.

Statement 2 is not correct: Bara Imambara is made of Lakhanui bricks, which are a bit small in size,
and lime plaster. No wood or metal has been used in the construction (except for the galleries in the
interior). This skill can be seen in the delicately built Rumi Darwaza that was the main gateway to the
Bara Imambara. It was called so because the design of the structure bears resemblance to an ancient
gateway at Constantinople. It’s also called the “Turkish Gateway”. The word Rumi means Roman,
and the name was probably given due to the gateway’s design having traces of Roman architecture.

Q21. With reference to India history, who among the following is a future Buddha, yet to come to
save the world?
(a) Avalokiteshvara
(b) Lokesvara
(c) Maitreya
(d) Padmapain

Answer: (c)

Option (c) is correct: In Buddhist culture, Maitreya is a bodhisattva who will arrive on Earth in the
future, and save the world, in order to attain complete awareness and educate about the pure
dharma. According to the inscriptions, Maitreya will be an heir to the existing Buddha, Gautama
Buddha. The prediction of the appearance of Maitreya relates to a time in the future when the dharma
will have been neglected by most in the physical world.
Science & Technology

Q1. With reference to the Indian Regional Navigation Satellite System (IRNSS), consider the
following statements:

1. IRNSS has three satellites in geostationary and four satellites in geosynchronous orbits.
2. IRNSS covers the entire India and about 5500 sq. km beyond its borders.
3. India will have its own satellite navigation system with full global coverage by the middle of
2021.
Which of the statements given above is/are correct?

(a) 1 only
(b) 1 and 2 only
(c) 2 and 3 only
(d) None
Answer: (a)

IRNSS with an operational name of NAVIC (Navigation with Indian Constellation) is an independent
regional navigation satellite system being developed by India. It is designed to provide accurate
position information service to users in India as well as the region extending up to 1500 km from its
boundary, which is its primary service area.

Statement 1 is correct: Three of the seven satellites in the constellation are located in geostationary
orbit (GEO) and four in inclined geosynchronous orbit (GSO).

Statement 2 is not correct: It covers India and a region extending 1,500 km around it, with plans for
further extension. The system at present consists of a constellation of seven satellites, with two
additional satellites on the ground as stand-by.

Statement 3 is not correct: The constellation is already in orbit and the system was expected to be
operational from early 2018 after a system check.

Q2. Consider the following phenomena:

1. Light is affected by gravity.


2. The Universe is constantly expanding.
3. Matter warps its surrounding space-time.
Which of the above is/are the predictions/predictions of Albert Einstein's General Theory of Relativity,
often discussed in the media?

(a) 1 and 2 only


(b) 3 only
(c) 1 and 3 only
(d) 1, 2 and 3

Answer: (d)

Statement 1 is correct: Einstein’s General Theory of Relativity is based on how we think gravity
governs the behaviour of the Universe. According to Einstein’s theory, light, just like any other form
of matter, is affected by gravity. That is, the light also “falls” in a gravitational field.

Statement 2 is correct: Steady-state theory, in cosmology the universe is always expanding but
maintaining a constant average density, with matter being continuously created to form new stars
and galaxies at the same rate that old ones become unobservable as a consequence of their increasing
distance and velocity of recession.

Statement 3 is correct: Hubble Space Telescope captured the image of a phenomenon called Einstein
Ring. Einstein in his theory of general relativity had suggested that a massive object would warp space
and time.

Q3. With reference to the Genetically Modified mustard (GM mustard) developed in India, consider
the following statements:

1. GM mustard has the genes of a soil bacterium that give the plant the property of pest-
resistance to a wide variety of pests.
2. GM mustard has the genes that allow the plant cross-pollination and hybridization.
3. GM mustard has been developed jointly by the IARI and Punjab Agricultural University.
Which of the statements given above is/are correct?

(a) 1 and 3 only


(b) 2 only
(c) 2 and 3 only
(d) 1, 2 and 3
Answer: (b)
Genetically modified crops are those types of crops which have been modified genetically, might
include increasing nutrients, reducing the maturation time of the plant, yields, and stress tolerance,
and creating a plant that can withstand diseases, and heavier applications of pesticides and herbicides.

Statement 1 is not correct: DMH-11 is a Genetically Modified (GM) mustard, and it is also an
herbicide tolerant mustard. DMH -11 contains three genes viz. Bar gene, Barnase and Barstar sourced
from soil bacterium. The genes of a soil bacterium do not give the plant the property of pest-
resistance to a wide variety of pests.

Statement 2 is correct: GM Mustard is a perfect flower so it ‘self-pollinates’. GM mustard has the


genes that allow the plant cross-pollination and hybridization.

Statement 3 is not correct: GM Mustard has been developed by a team of scientists at Delhi
University (not IARI and Punjab University) led by former vice-chancellor Deepak Pental with the aim
of reducing India's demand for edible oil imports. They developed Dhara Mustard Hybrid (DMH) -11,
a genetically modified hybrid variety of the mustard species (Brassica juncea).

Q4. Consider the following pairs:

Terms sometimes seen in news Context/Topic

1. Belle II experiment :Artificial Intelligence


2. Blockchain technology :Digital/Cryptocurrency
3. CRISPR-Cas9 :Particle Physics

Which of the pairs given above is/are correctly matched?

(a) 1 and 3 only


(b) 2 only
(c) 2 and 3 only
(d) 1, 2 and 3
Answer: (b)

Pair 1 is not correctly matched: The Belle II experiment is a particle physics experiment designed to
study the properties of B mesons (heavy particles containing a bottom quark).

Pair 2 is correctly matched: Blockchain is a system which helps in recording information. The
information is recorded in such a way that it makes it difficult for anyone to hack or cheat the system.
Unlike physical currencies, blockchain cryptocurrency works on digital channels and are often
adhered to strong cryptography to secure financial transactions that happen online. Blockchain
technology - Digital/ Cryptocurrency.

Pair 3 is not correctly matched: CRISPR- Cas9 is a Gene-editing tool that can be used to modify DNA
in cells. CRISPR-Cas9 was adapted from a naturally occurring genome editing system in bacteria. The
bacteria capture snippets of DNA from invading viruses and use them to create DNA segments known
as CRISPR arrays. The CRISPR arrays allow the bacteria to "remember" the viruses (or closely related
ones).

Q5. When the alarm of your smartphone rings in the morning, you wake up and tap it to stop the
alarm which causes your geyser to be switched on automatically. The smart mirror in your bathroom
shows the day's weather and also indicates the level of water in your overhead tank. After you take
some groceries from your refrigerator for making breakfast, it recognises the shortage of stock in it
and places an order for the supply of fresh grocery items. When you step out of your house and lock
the door, all lights, fans, geysers and AC machines get switched off automatically. On your way to
the office, your car warns you about traffic congestion ahead and suggests an alternative route, and
if you are late for a meeting, it sends a- message to your office accordingly.

In the context of emerging communication technologies, which one of the following terms best
applies to the above scenario?

(a) Border Gateway Protocol


(b) Internet of Things
(c) Internet Protocol
(d) Virtual Private Network

Answer: (b)

Option (b) is correct: Internet of Things (IoT) the interconnection via the internet of computing
devices embedded in everyday objects, enabling them to send and receive data.

It is the wireless sensor network (WSN) of physical devices, vehicles, home appliances and other
items embedded with electronics, software, sensors, actuators, and connectivity which enables
these things to connect and exchange data, creating opportunities for more direct integration of the
physical world into computer-based systems, resulting in efficiency improvements, economic benefits
and reduced human intervention.

Q6. In which of the following areas, can GPS technology be used?


1. Mobile phone operations
2. Banking operations
3. Controlling the power grids
Select the correct answer using the codes given below:

(a) 1 only
(b) 2 and 3 only
(c) 1 and 3 only
(d) 1, 2 and 3

Answer: (d)

Global Positioning System (GPS) is a satellite-based navigation system made up of at least 24


satellites covering Earth from space. Core Applications of GPS technology are Navigation, Timing,
Tracking, Mapping and Location.

Option (d) is correct: GPS is used for mobile phone operations (Example: Cell towers use it to route
phone calls), banking operations (Example: ATMs and cash registers use it for transactions) and
controlling the power grids (Example: electrical grids use it to send power to house).

GPS is also used to identify, locate, and maintain contact reports with one or more fleet vehicles in
real-time. Astronomy, Automated vehicle, Cartography, Clock synchronization, Disaster
relief/emergency services, Radio occultation for weather and atmospheric science applications.

Q7. The terms ‘Wanna Cry, Petya and Eternal Blue' sometimes mentioned in the news recently are
related to

(a) Exoplanets
(b) Crypto currency
(c) Cyber attacks
(d) Minisatellites
Answer: (c)

Option (c) is correct: WannaCry is a ransomware which is used for cyberattacks worldwide. It targets
computers running the Microsoft Windows operating system by encrypting data and demanding
ransom payments in Bitcoin cryptocurrency.
The Petya ransomware not only encrypts files, but it also locks the entire disk, making it basically
unusable until the infection is removed. It shuts down the system and asks for a ransom of $300 in
bitcoins on rebooting.

Code known as “Eternal Blue,” which cyber security experts widely believe was stolen from the US
National Security Agency (NSA) and was also used in a ransomware attack, named “WannaCry.”

Q8. "3D printing" has applications in which of the following?

1. Preparation of confectionery items


2. Manufacture of bionic ears
3. Automotive industry
4. Reconstructive surgeries
5. Data processing technologies
Select the correct answer using the code given below:

(a) 1, 3 and 4 only


(b) 2, 3 and 5 only
(c) 1 and 4 only
(d) 1, 2, 3, 4 and 5

Answer: (d)

3D printing is additive manufacturing technology where a 3D object is created by laying down


successive layers of material. From construction to medical technology, 3D printing is being used
across a range of industries. Its use in the food industry continues to grow.

Option (d) is correct: 3D printing technology has applications in the preparations of confectionery
items, focused primarily on the chocolate and hard sugar candy industry. 3D printed bionic ears were
generated via 3D printing of a cell-seeded hydrogel matrix in the anatomic geometry of a human ear
in America. Industrial 3D printers have opened new paths at each stage of the production of motor
vehicles in the automotive industry. It has been used in reconstructive surgery to help repair
damaged tissue; for example - in cases of birth defects. 3D scanning is the process of analysing a real-
world object or environment to collect data on its shape and possibly its appearance (e.g., colour) then
the collected data can be used to Construct or Print digital 3D models, hence, it is useful in the data
processing technologies as well.
Q9. The identity platform `Aadhaar' provides open "Application Programming Interfaces (APIs)".
What does it imply?

1. It can be integrated into any electronic device.


2. Online authentication using iris is possible.
Which of the statements given above is/are correct?

(a) 1 only
(b) 2 only
(c) Both 1 and 2
(d) Neither 1 nor 2

Answer: (c)

Application Programming Interface (API) is Computer Program code that lets two applications
interact with each other. It helps in letting software exchange data and use each other’s capabilities.

Statement 1 is correct: Aadhaar provides open Application Programming Interfaces, which can be
integrated easily into any electronic device. Its open application programming interface (API) layers
known as “India Stack” set Aadhaar apart from other biometric ID programmes. India Stack APIs,
which include the Unified Payment Interface (UPI) and Aadhaar e-KYC, allow applications to be built
on a top App (for example, the Bharat Interface for Money or BHIM app) and enable identity-driven
transactions.

Statement 2 is correct: The Aadhaar authentication Application programming interface provides


various ways for an Aadhaar holder to authenticate themselves. At a higher level, this authentication
takes place using Demographics data or Biometric data, and OTP. Moreover, online authentication
using iris is also possible.

Q10. With reference to India’s satellite launch vehicles, consider the following statements:

1. PSLVs launch the satellites useful for Earth resources monitoring whereas GSLVs are designed
mainly to launch communication satellites.
2. Satellites launched by PSLV appear to remain permanently fixed in the same position in the
sky, as viewed from a particular location on Earth.
3. GSLV Mk III is a four-staged launch vehicle with the first and third stages l using solid rocket
motors; and the second and fourth stages using liquid rocket engines.
Which of the statements given above is/are correct?

(a) 1 only
(b) 2 and 3 only
(c) 1 and 2 only
(d) 3 only
Answer: (a)

To develop the technology needed to launch satellites, Satellite Launch Vehicle or SLV, a small lift
launch vehicle project started in the early 1970s by the Indian Space Research Organization.

Statement 1 is correct: The GSLV is designed mainly to deliver the communication-satellites to the
highly elliptical Geosynchronous Transfer Orbit (GTO) whereas PSLVs launch the satellites useful for
Earth resources monitoring.

Statement 2 is not correct: PSLV is designed mainly to deliver the “earth-observation” or “remote-
sensing” satellites with lift-off mass of up to about 1750 Kg to Sun-Synchronous circular polar orbits
of 600-900 Km altitude. Satellites launched by PSLV do not appear to remain permanently fixed in
the same position in the sky, as viewed from a particular location on Earth.
Statement 3 is not correct: GSLV Mk III is a three-stage heavy lift (not four stage) launch vehicle
developed by ISRO. First stage using solid fuel, the second stage liquid fuel and the third stage
cryogenic technology with the use of liquid hydrogen and oxygen.

Q11. What is "Terminal High Altitude Area Defense (THAAD)", sometimes seen in the news?

(a) An Israeli radar system


(b) India's indigenous anti-missile program
(c) An American anti-missile system
(d) A defence collaboration between Japan and South Korea
Answer: (c)

Option (c) is correct: THAAD is a relatively recent addition to the United States’ anti-ballistic
missile/interceptor toolkit. THAAD is particularly well-suited to intercept and destroy short, medium,
and intermediate-range ballistic missiles in their terminal phase.

THAAD is able to intercept incoming missiles at endo- and exo-atmospheric altitudes, with a
maximum engagement altitude of roughly 93 miles above the earth’s surface. In South Korea, the
THAAD missile defence system is operated by the US army stationed in the country. The US had
previously announced that the deployment of this missile defence system was a countermeasure
against potential attacks by North Korea, particularly after the country had engaged in testing ballistic
missiles.

Current Affairs

Q1. Consider the following statements:

1. Aadhaar card can be used as a proof of citizenship or domicile.


2. Once issued, Aadhaar number cannot be deactivated or omitted by the Issuing Authority.

Which of the statements given above is/are correct?

(a) 1 only
(b) 2 only
(c) Both 1 and 2
(d) Neither 1 nor 2

Answer: (d)

Statement 1 is not correct: Section 9 of the Aadhaar Act states that “The Aadhaar number or the
authentication, thereof shall not, by itself, confer any right of, or be proof of, citizenship or domicile
in respect of an Aadhaar number holder.” Conversely, it implies that citizenship itself does not imply
that an Aadhaar card has been validly obtained.

Statement 2 is not correct: The Unique Identification Authority of India (UIDAI) on its website
mentioned following provisions for deactivation of Aadhaar: If within 2 years of attaining age 5, the
child’s biometrics are not updated in the Aadhaar database, his/her Aadhaar number will be
deactivated (no authentication permissible). It will be reactivated once biometrics are updated in the
database. If a resident has not been biometrically authenticated in 5 years, his/her One Time Password
(OTP) based authentication services will be deactivated. They will be reactivated once the resident is
biometrically authenticated.

Q2. “Rule of Law Index” is released by which of the following?

(a) Amnesty International


(b) International Court of Justice
(c) The Office of UN Commissioner for Human Rights
(d) World Justice Project

Answer:(d)

Option (d) is correct: The Rule of Law Index is released by the World Justice Project. The Index is a
quantitative assessment tool designed to offer a comprehensive picture of the extent to which
countries adhere to the rule of law. The scores of the Index can be interpreted as degrees of
compliance of a legal system to a definition of Rule of Law.

The index measures countries’ rule of law performance across eight factors: Constraints on
Government Powers, Absence of Corruption, Open Government, Fundamental Rights, Order and
Security, Regulatory Enforcement, Civil Justice, and Criminal Justice.

Q3. Very recently, in which of the following countries having lakhs of people either suffered from
severe famine/acute malnutrition or died due to starvation caused by war/ethnic conflicts?

(a) Angola and Zambia


(b) Morocco and Tunisia
(c) Venezuela and Colombia
(d) Yemen and South Sudan

Answer: (d)

The United Nations (UN) defines famine as a crisis where at least 20 percent of a region’s population
does not have sufficient food to be healthy, more than 30 percent of children under the age of five
are severely malnourished, and two in 10,000 people or four in 10,000 children die due to extreme
hunger every day.

Option (d) is correct: Presently, around 20 million people, including 1.4 million children, are inching
closer to this state of living as the spectre of famine looms large over South Sudan, Somalia, Northeast
Nigeria, and Yemen. People in these regions either suffered from severe famine/acute malnutrition
or died due to starvation caused by war/ethnic conflicts.

It is a man-made famine, stemming from rapacious power struggles, is threatening the lives of
innocent civilians, especially children, who, even if they survive, are likely to be mentally or physically
stunted for the rest of their lives.

Q4. The term “Two state Solution” is sometimes mentioned in the new in the context of the affairs
of

(a) China
(b) Israel
(c) Iraq
(d) Yemen

Answer: (b)

Option (b) is correct: The two-state solution would establish an independent Palestinian state
alongside Israel — two states for two peoples. This is a proposed framework for resolving the Israeli-
Palestinian conflict by establishing two states for two peoples: Israel for the Jewish people and
Palestine for the Palestinian people. In 1993 the Israeli government and the Palestine Liberation
Organization (PLO) agreed on a plan to implement a two-state solution as part of the Oslo Accords,
leading to the establishment of the Palestinian Authority (PA).

Q5. In the Indian context, what is the implication of ratifying the ‘Additional Protocol’ with the
‘International Atomic Energy Agency (IAEA)’?

(a) The civilian nuclear reactors come under IAEA safeguards.


(b) The military nuclear installations come under the inspection of IAEA.
(c) The country will have the privilege to buy uranium from the Nuclear Suppliers Group (NSG).
(d) The country automatically becomes a member of the NSG.

Answer: (a)

The International Atomic Energy Agency (IAEA) headquartered in Vienna, Austria. It is the world's
centre for cooperation in the nuclear field and seeks to promote the safe, secure, and peaceful use
of nuclear technologies. It was established as an autonomous organisation on 29 July 1957. The IAEA
reports to both the United Nations General Assembly and the Security Council.

Option (a) is correct: The Additional Protocol is not a stand-alone agreement, but rather a protocol
to a safeguards agreement that provides additional tools for verification. In particular, it significantly
increases the IAEA’s ability to verify the peaceful use of all nuclear material in States with
comprehensive safeguards agreements. Hence, the civilian nuclear reactors come under IAEA
safeguards

An Additional Protocol to the Safeguards Agreement between the Government of India and the IAEA
for the Application of Safeguards to Civilian Nuclear Facilities entered into force on 25 July 2014.

Q6. With reference to solar power production in India, consider the following statements:
1. India is the third largest in the world in the manufacture of silicon wafers used in photovoltaic
units.

2. The solar power tariffs are determined by the Solar Energy Corporation of India.

Which of the statements given above is/are correct?

(a) 1 only
(b) 2 only
(c) Both 1 and 2
(d) Neither 1 nor 2

Answer: (d)

Silicon is a chemical element and a semiconductor that is economically important worldwide due to
its industrial uses in steel refining, aluminium-casting, chemical processes, and perhaps as it is most
widely known, in semiconductor electronics. Silicon wafers are thin slices of semiconductor that are
used for the fabrication of integrated circuits, and to manufacture solar cells.

Statement 1 is not correct: China is the world’s largest silicon producer, with a production volume
estimated at 5.4 million metric tons in 2020. The second largest producer of this metalloid in the world
is Russia, Norway being third followed by the United States and brazil. Every solar panel which is
made in India is assembled while all the material comes from China, Europe and some other
countries

Statement 2 is not correct: Central Electricity Regulatory Commission determines power tariffs
including solar power tariffs in India and regulates the tariff of generating companies owned or
controlled by the Central Government. Solar Energy Corporation of India has a power-trading license,
but it does not set solar power tariffs.

Q7. Consider the following statements:

1. As per the right to education (RTE) Act, to be eligible for appointment as a teacher in a state,
a person would be required to possess the minimum qualification laid down by the concerned
State Council of Teacher education.
2. As per the RTE Act, for teaching primary classes, a candidate is required to pass a Teacher
Eligibility Test conducted in accordance with the National Council of Teacher Education
guidelines.
3. In India, more than 90% of teacher education institutions are directly under the State
Governments.

Which of the statements given above is/are correct?

a) 1 and 2 only
b) 2 only
c) 1 and 3 only
d) 3 only

Answer: (b)

Statement 1 is not correct: In 2011, the National Council for Teacher Education (NCTE) laid down the
teacher recruitment norms for all government schools in India irrespective of the State. According
to it, the stipulated qualification for teaching Class 6 to Class 8 is either a four-year Bachelor of
Elementary Education or a B.Ed. degree after senior secondary; or a two-year B.Ed. after graduation;
or a two-year Diploma in Education (D.Ed.) plus a graduate degree.

Statement 2 is correct: The Teacher Eligibility Test, known as TET, is the minimum qualification
required in India for a person to be eligible for appointment as a teacher for Classes I to VIII. The test
is mandatory for teaching jobs in Indian government schools.

Statement 3 is not correct: The NCTE is the apex body that manages the teacher’s education
institutions in the country. The key role of the NCTE is to ‘achieve planned and coordinated
development of the teacher education system throughout the country. However, more than 90% of
the teacher’s education institutions across the country are privately managed.

Q8. Consider the following statements:

1. In India, State Governments do not have the power to auction non-coal mines.
2. Andhra Pradesh and Jharkhand do not have gold mines.
3. Rajasthan has iron ore mines.

Which of the statements given above is/are correct?

(a) 1 and 2
(b) 2 only
(c) 1 and 3
(d) 3 only
Answer: (d)

Statement 1 is not correct: As per the Mines and Minerals (Development and Regulation)
Amendment Act, 2015 the non-coal mines have to be auctioned by the respective State governments.

Statement 2 is not correct: The largest resources in terms of gold ore (primary) are located in Bihar
(44%) followed by Rajasthan (25%), Karnataka (21%), West Bengal (3%), Andhra Pradesh (3%),
Jharkhand (2 %). The remaining 2% resources of ore are in Chhattisgarh, Madhya Pradesh, Kerala,
Maharashtra, and Tamil Nadu. Andhra Pradesh and Jharkhand do have gold mines.

Statement 3 is correct: About 2621 million tons of resources of Iron–Ore (both Hematite &
Magnetite) are estimated in Rajasthan. The Iron-Ore deposits are located in the district of Jaipur,
Udaipur, Jhunjhunu, Sikar, Bhilwara, Alwar, Bharatpur, Dausa and Banswara. The important localities
of Iron-Ore are Morija- Neemala (Jaipur), Lalsot (Dausa), Rampura, Dabla (Sikar), Taonda (Jhunjhunu),
Pur-Banera, Bigod (Bhilwara), Nathara-Ki-Pal, Thur (Udaipur), Indergarh, Mohanpura (Bundi),
Dedrauli, Liloti, Todupura, Khora (Karauli).

Q9. Consider the following:

1. Areca nut

2. Barley

3. Coffee

4. Finger millet

5. Groundnut

6. Sesamum

7. Turmeric

The Cabinet Committee on Economic Affairs has announced the Minimum Support Price for which of
the above?

(a) 1, 2, 3 and 7 only


(b) 2, 4, 5 and 6 only
(c) 1, 3, 4, 5 and 6 only
(d) 1, 2, 3, 4, 5 and 7
Answer: (b)

Minimum Support Price (MSP) is a form of market intervention by the Government of India to insure
agricultural producers against any sharp fall in farm prices. The minimum support prices are
announced by the The Cabinet Committee on Economic Affairs, Government of India at the beginning
of the sowing season for certain crops on the basis of the recommendations of the Commission for
Agricultural Costs and Prices (CACP).

Option (b) is correct: Government announces MSP for 22 mandated crops and fair and remunerative
price (FRP) for sugarcane. The mandated crops are 14 crops of the kharif season, 6 rabi crops and two
other commercial crops. The list of crops are as follows:

Cereals: Paddy, Wheat, Barley, Jowar, Bajra, Maize and Ragi (finger millet).

Pulses: Gram, Arhar/tur, Moong, Urad and Lentil.

Oilseeds: Groundnut, Rapeseed/mustard, Toria, Soyabean, Sunflower seed, Sesamum, Safflower seed
and Niger seed.

Others: Raw cotton, Raw jute, Copra, De-husked coconut, Sugarcane and Virginia flu cured (VFC)
tobacco.

Q10. Which of the following is/are the aims of “Digital India” Plan of the Government of India?

1. Formation of India’s own Internet companies like China did.


2. Establish a policy framework to encourage overseas multinational corporations that collect Big
Data to build their large data centres within our national geographical boundaries.
3. Connect many of our villages to the Internet and bring Wi-Fi to many of our schools, public
places and major tourist centres.

Select the correct answer using the code given below:

(a) 1 and 2 only


(b) 3 only
(c) 2 and 3 only
(d) 1, 2 and 3

Answer: (b)
Digital India campaign was launched in July 2015, with the aim of ensuring that the government's
services are made available to citizens electronically by improving online infrastructure and by
increasing Internet connectivity or making the country digitally empowered in the field of technology.

Option (b) is correct: The initiative includes connecting many of our villages to the internet and
bringing Wi-Fi to many of our schools, public places, and major tourist centres. Digital India consists
of three main components: The development of secure and stable digital infrastructure, Delivering
government services digitally, and Universal digital literacy.

Q11. Which is /are the consequence / consequences of becoming a member of the ‘Nuclear Supplier
Group’?

1. It will have access to the latest and most effective nuclear technologies.
2. It automatically becomes a member of “The Treaty of non- proliferation of Nuclear weapons (NPT)”.
Which of the statements given below is/ are correct?

(a) 1 only
(b) 2 only
(c) Both 1 and 2
(d) Neither 1 nor 2

Answer: (a)

The Nuclear Suppliers Group (NSG) is a group of nuclear supplier countries that seeks to contribute
to the non-proliferation of nuclear weapons through the implementation of two sets of guidelines for
nuclear exports and nuclear-related exports.

Statement 1 is correct: All nuclear based programmes of India are being run on indigenous
technology. By becoming a member of the Nuclear Supplier Group (NSG), India will have access to
the latest and most effective nuclear technologies. If India becomes a member, it will also have a
better international market for export as well as for import of nuclear related materials.

Statement 2 is not correct: NPT (Non-proliferation Treaty) is an international treaty, which came into
force in 1970. The main objective was to prevent the spread of nuclear weapons and weapons
technology. The nuclear powers were convinced that the Nuclear Non-Proliferation Treaty (NPT) alone
would not halt the spread of nuclear weapons. Also, joining the NSG will not automatically lead to
becoming a member of “The Treaty of non- proliferation of Nuclear weapons (NPT). In a similar case
in 2008, India had got a ‘clean waiver’ from NSG to do nuclear trade with member countries, where
in, it is forbidden to trade with a country who has not signed the NPT.

You might also like